Google Groups no longer supports new Usenet posts or subscriptions. Historical content remains viewable.
Dismiss

Einstein's 1939 paper challenging the existence of black holes

93 views
Skip to first unread message

Melroy

unread,
Apr 27, 2009, 1:46:40 AM4/27/09
to
Hi,
From watching this talk http://online.kitp.ucsb.edu/online/joint98/mottola/
it seems that Einstein and Dirac has strong issues against the
existence of black holes
and had published stuff in 1939 and 1962 respectively arguing against
black holes.
But this was before the work by Penrose in 60s.
Einstein's 1939 paper is published in Ann. of Math 40, 922
Can some experts (Jonathan Thornburg, Steve Carlip etc) point to me
the mistake in Einstein's 1939 paper and how
the current understanding of the picture on gravitational collapse
circumvents Einstein's arguments in his 1939 paper. I have not seen
much discussion of these papers on arxiv or in this newsgroup. Thanks
very much

Oh No

unread,
Apr 27, 2009, 11:47:23 AM4/27/09
to
Thus spake Melroy <melroy...@hotmail.com>

Thanks for these references.

I think the more relevant quote from Einstein is this one:

"Besides, the choice of such an equation of state would be arbitrary
within very wide limits, and one could not be sure that thereby no
assumptions have been made which contain physical impossibilities".


I particularly like this from Dirac:

"The mathematicians can go beyond the Schwarzschild radius, and get
inside, but I would maintain that this region is not physical space,
because to send a signal inside and get it out again would take an
infinite time, so I feel that the space inside the Schwarzschild radius
must belong to a different universe and should not be taken into account
in any physical theory. So, from the physical point of view, the
possibility of having a point singularity in the Einstein field is ruled
out. Each particle must have a finite size no smaller than the
Schwarzschild radius.

I tried for some time to work with a particle with radius equal to the
Schwarzschild radius, but I found great difficulties, because the field
at the Schwarzschild radius is so strongly singular, and it seems that a
more profitable line of investigation is to take a particle bigger than
the Schwarzschild radius, and to try to construct a theory for such a
particle interacting with the gravitational field."

Proc. R. Soc. Lond. A November 27, 1962 270:354-356;
doi:10.1098/rspa.1962.0228

Einstein originally had similar problems with Friedmann's solutions to
the field equation, though he did come to accept them. Equations are
just equations. If one pushes solution to extreme situations, then one
must question whether the equations still apply.

In practice mathematicians sometimes claim that actually spacetime is
not singular, or even particularly extreme at the Schwarzschild radius.
Nonetheless, I think Dirac's argument is very sound.

The principle arguments for black holes are a) Chandrasekhar's, which
shows that according to our understanding of particle physics, fluids of
sufficient mass do become infinitely compressible. b) Openheimer and
Snyder's, who showed equations resulting in black hole formation, and c)
Penrose & Hawking's singularity theorems, showing the necessity of
singularities.

The counter argument, to which Einstein would I think have subscribed,
is that the equations can reasonably be expected to break down prior to
the singularity itself.

Observation is that black holes exist, at least to the extent that the
description of a black hole is valid outside the Schwarzschild radius.
That is to say we observe objects which we can explain as black holes,
and we observe that the density in the centre of the galaxy is
sufficiently high that we can only describe it as a black hole.

My own research has followed a line of thought very similar to that
described by Dirac. (I always find it encouraging to discover that a
great physicist has thought along lines I have followed myself :-) ).
However, I have found a solution to the problem of using particles with
radius equal to the Schwarzschild radius, by using particular
coordinates in which the (radial) speed of light is unity. In these
coordinates, the Schwarzschild radius appears at r = 0, and there is no
region inside. I give a description at

http://rqgravity.net/OriginOfCurvature

Regards

--
Charles Francis
moderator sci.physics.foundations.
charles (dot) e (dot) h (dot) francis (at) googlemail.com (remove spaces and
braces)

http://www.rqgravity.net/MainIndex

Phillip Helbig---remove CLOTHES to reply

unread,
Apr 27, 2009, 5:28:46 PM4/27/09
to
In article <nJKFHZFw...@charlesfrancis.wanadoo.co.uk>, Oh No
<No...@charlesfrancis.wanadoo.co.uk> writes:

> In practice mathematicians sometimes claim that actually spacetime is
> not singular, or even particularly extreme at the Schwarzschild radius.
> Nonetheless, I think Dirac's argument is very sound.

If you make a black hole arbitrarily massive, then the tidal effects etc
at the Schwarzschild radius become arbitrarily small. Certainly in such
cases spacetime is not "extreme". What do you mean by "sometimes"? Are
there mathematicians who claim that "spacetime is extreme" at the
Schwarzschild radius (whatever its value)?

> My own research has followed a line of thought very similar to that
> described by Dirac. (I always find it encouraging to discover that a
> great physicist has thought along lines I have followed myself :-) ).

"All my best thoughts were stolen by the ancients." (Including this
one---by Ralph Waldo Emerson.)

Oh No

unread,
Apr 27, 2009, 6:37:14 PM4/27/09
to
Thus spake Phillip Helbig---remove CLOTHES to reply <hel...@astro.multiC
LOTHESvax.de>

>In article <nJKFHZFw...@charlesfrancis.wanadoo.co.uk>, Oh No
><No...@charlesfrancis.wanadoo.co.uk> writes:
>
>> In practice mathematicians sometimes claim that actually spacetime is
>> not singular, or even particularly extreme at the Schwarzschild radius.
>> Nonetheless, I think Dirac's argument is very sound.
>
>If you make a black hole arbitrarily massive, then the tidal effects etc
>at the Schwarzschild radius become arbitrarily small. Certainly in such
>cases spacetime is not "extreme". What do you mean by "sometimes"?

I mean that mathematicians sometimes say this, not that the results of a
determinist calculation can sometimes be any different :-)

Eric Gisse

unread,
Apr 28, 2009, 12:15:07 PM4/28/09
to
On Apr 27, 7:47�am, Oh No <N...@charlesfrancis.wanadoo.co.uk> wrote:

[...]

> I particularly like this from Dirac:
>
> "The mathematicians can go beyond the Schwarzschild radius, and get
> inside, but I would maintain that this region is not physical space,
> because to send a signal inside and get it out again would take an
> infinite time, so I feel that the space inside the Schwarzschild radius
> must belong to a different universe and should not be taken into account
> in any physical theory. So, from the physical point of view, the
> possibility of having a point singularity in the Einstein field is ruled
> out. Each particle must have a finite size no smaller than the
> Schwarzschild radius.

Note how this predates the understanding of event horizons in general
relativity.

The arguments end up all revolving around either the Schwarzschild
radius or the physicality of the solution. That black holes were /
physically/ possible was worked out by Oppenheimer and Chandresakhar.

On the other hand, the Schwarzschild radius wasn't shown to be an
artifact until Kruskal discovered the coordinate chart that smoothly
connected the inner and outer regions of a black hole. That didn't
happen until the early 60's, as I recall. Pretty close to the
publication of this, but I have no idea if Dirac knew of the work or
if the work pre-dated this - even if it did it wasn't by too long.

>
> I tried for some time to work with a particle with radius equal to the
> Schwarzschild radius, but I found great difficulties, because the field
> at the Schwarzschild radius is so strongly singular, and it seems that a
> more profitable line of investigation is to take a particle bigger than
> the Schwarzschild radius, and to try to construct a theory for such a
> particle interacting with the gravitational field."
>
> Proc. R. Soc. Lond. A November 27, 1962 270:354-356;
> doi:10.1098/rspa.1962.0228
>
> Einstein originally had similar problems with Friedmann's solutions to
> the field equation, though he did come to accept them. �Equations are
> just equations. If one pushes solution to extreme situations, then one
> must question whether the equations still apply.

That's where most of the argument comes from. To this day, even.

So far so good, it looks. Nobody knows what's inside a black hole and
even though there are some people I'd willingly have try a look-see,
we'd still not know.

>
> In practice mathematicians sometimes claim that actually spacetime is
> not singular, or even particularly extreme at the Schwarzschild radius.
> Nonetheless, I think Dirac's argument is very sound.

With respect to Dirac, it isn't.

The singular-ness of the event horizon is a coordinate artifact, which
in everyone's defense, did take a half century of work to understand.

>
> The principle arguments for black holes are a) Chandrasekhar's, which
> shows that according to our understanding of particle physics, fluids of
> sufficient mass do become infinitely compressible. b) Openheimer and
> Snyder's, who showed equations resulting in black hole formation, and c)
> Penrose & Hawking's singularity theorems, showing the necessity of
> singularities.

d) Buchdhal's theorem. Under spherical symmetry, hydrostatic
equilibrium, and reasonable (physical) energy conditions, it is
impossible to stuff more than a certain amount of mass in a given
sized sphere without the central pressure becoming infinite.

Also, Dirac's argument predates the singularity theorems by a solid
decade.

>
> The counter argument, to which Einstein would I think have subscribed,
> is that the equations can reasonably be expected to break down prior to
> the singularity itself.

Entirely possible and most likely true, but any breakage past the
Schwarzschild radius is unobservable and is a moot point.

A few more theorems - also by Hawking & Penrose - speak of black hole
hair. Take into account the no-hair theorems which highly restrict
what kind of fields can extend past the event horizon, as well as
Birkhoff's theorem which makes a strong statement about the exterior
solution of any spherically symmetric mass, that at some limit GR has
to be retained, and it should be realized that any deviation is most
likely contained within the event horizon barring a severe departure
from GR.

[...]

Bob_for_short

unread,
Apr 28, 2009, 11:16:03 PM4/28/09
to
On 27 avr, 07:46, Melroy <melroysoa...@hotmail.com> wrote:
> Hi,
> From watching this talkhttp://online.kitp.ucsb.edu/online/joint98/mottola/

I read recently that the Schwarzschild radius cannot exceed a body
radius, otherwise one needs singular coordinate transformation to
match solutions "in" and "out" the sphere (Logunov et al.). Singular
coordinate transformations are not physical and should not be used.

As soon as any material body is larger than the Schwarzschild radius,
it is not a black hole anymore.

Bob.

Oh No

unread,
Apr 28, 2009, 11:17:10 PM4/28/09
to
Thus spake Eric Gisse <jow...@gmail.com>

>On Apr 27, 7:47 am, Oh No <N...@charlesfrancis.wanadoo.co.uk> wrote:
>
>[...]
>
>> I particularly like this from Dirac:
>>
>> "The mathematicians can go beyond the Schwarzschild radius, and get
>> inside, but I would maintain that this region is not physical space,
>> because to send a signal inside and get it out again would take an
>> infinite time, so I feel that the space inside the Schwarzschild radius
>> must belong to a different universe and should not be taken into account
>> in any physical theory. So, from the physical point of view, the
>> possibility of having a point singularity in the Einstein field is ruled
>> out. Each particle must have a finite size no smaller than the
>> Schwarzschild radius.
>
>Note how this predates the understanding of event horizons in general
>relativity.
>
>The arguments end up all revolving around either the Schwarzschild
>radius or the physicality of the solution. That black holes were /
>physically/ possible was worked out by Oppenheimer and Chandresakhar.
>
>On the other hand, the Schwarzschild radius wasn't shown to be an
>artifact until Kruskal discovered the coordinate chart that smoothly
>connected the inner and outer regions of a black hole. That didn't
>happen until the early 60's, as I recall. Pretty close to the
>publication of this, but I have no idea if Dirac knew of the work or
>if the work pre-dated this - even if it did it wasn't by too long.

Dirac was 1962, Kruskal-Szekeres coordinates were found in 1960. Dirac
may not have known of them, but I think Eddington-Finkelstein
coordinates were probably earlier, and I suspect Dirac was referring to
either Kruskal-Szekeres or Eddington-Finkelstein when he said "The


mathematicians can go beyond the Schwarzschild radius"
>
>>

>> I tried for some time to work with a particle with radius equal to the
>> Schwarzschild radius, but I found great difficulties, because the field
>> at the Schwarzschild radius is so strongly singular, and it seems that a
>> more profitable line of investigation is to take a particle bigger than
>> the Schwarzschild radius, and to try to construct a theory for such a
>> particle interacting with the gravitational field."
>>
>> Proc. R. Soc. Lond. A November 27, 1962 270:354-356;
>> doi:10.1098/rspa.1962.0228
>>
>> Einstein originally had similar problems with Friedmann's solutions to
>> the field equation, though he did come to accept them.  Equations are
>> just equations. If one pushes solution to extreme situations, then one
>> must question whether the equations still apply.
>
>That's where most of the argument comes from. To this day, even.
>
>So far so good, it looks. Nobody knows what's inside a black hole and
>even though there are some people I'd willingly have try a look-see,
>we'd still not know.

indeed.

>> In practice mathematicians sometimes claim that actually spacetime is
>> not singular, or even particularly extreme at the Schwarzschild radius.
>> Nonetheless, I think Dirac's argument is very sound.
>
>With respect to Dirac, it isn't.
>
>The singular-ness of the event horizon is a coordinate artifact, which
>in everyone's defense, did take a half century of work to understand.

Yes, but this does not mean that there is no breakdown in the physics,
past which the equations do not apply.

>> The principle arguments for black holes are a) Chandrasekhar's, which
>> shows that according to our understanding of particle physics, fluids of
>> sufficient mass do become infinitely compressible. b) Openheimer and
>> Snyder's, who showed equations resulting in black hole formation, and c)
>> Penrose & Hawking's singularity theorems, showing the necessity of
>> singularities.
>
>d) Buchdhal's theorem. Under spherical symmetry, hydrostatic
>equilibrium, and reasonable (physical) energy conditions, it is
>impossible to stuff more than a certain amount of mass in a given
>sized sphere without the central pressure becoming infinite.
>
>Also, Dirac's argument predates the singularity theorems by a solid
>decade.

Yes, but the singularity theorems are not proof against a physical
limit.

>> The counter argument, to which Einstein would I think have subscribed,
>> is that the equations can reasonably be expected to break down prior to
>> the singularity itself.
>
>Entirely possible and most likely true, but any breakage past the
>Schwarzschild radius is unobservable and is a moot point.
>
>A few more theorems - also by Hawking & Penrose - speak of black hole
>hair. Take into account the no-hair theorems which highly restrict
>what kind of fields can extend past the event horizon, as well as
>Birkhoff's theorem which makes a strong statement about the exterior
>solution of any spherically symmetric mass, that at some limit GR has
>to be retained, and it should be realized that any deviation is most
>likely contained within the event horizon barring a severe departure
>from GR.
>

Again, these theorems require that the manifold is physically
meaningful. The reason I believe the breakdown is actually at the
Schwarzschild radius is because the coordinates I use are such that I
can define quantum theory consistent with gtr, and because in these
coordinates the Schwarzschild radius is at r=0, so there simply is no
inside.

juanR...@canonicalscience.com

unread,
Apr 29, 2009, 6:21:30 AM4/29/09
to
Eric Gisse wrote on Tue, 28 Apr 2009 18:15:07 +0200:

(...)

> On the other hand, the Schwarzschild radius wasn't shown to be an
> artifact until Kruskal discovered the coordinate chart that smoothly
> connected the inner and outer regions of a black hole.

"Schwarzschild radius" is not an artifact, just a radius.

The Kruskal Szekres coordinates show that spacetime metric is not
singular at the "Schwarzschild radius", just that.

Of course, the Kruskal Szekres coordinates continues containing the event
horizon corresponding to the "Schwarzschild radius".

(...)

--=20
http://www.canonicalscience.org/

Usenet Guidelines:
http://www.canonicalscience.org/en/miscellaneouszone/guidelines.html

Gerry Quinn

unread,
Apr 30, 2009, 6:40:56 AM4/30/09
to
In article <gt56ju$rnh$1...@online.de>, hel...@astro.multiCLOTHESvax.de
says...

> In article <nJKFHZFw...@charlesfrancis.wanadoo.co.uk>, Oh No
> <No...@charlesfrancis.wanadoo.co.uk> writes:
>
> > In practice mathematicians sometimes claim that actually spacetime is
> > not singular, or even particularly extreme at the Schwarzschild radius.
> > Nonetheless, I think Dirac's argument is very sound.
>
> If you make a black hole arbitrarily massive, then the tidal effects etc
> at the Schwarzschild radius become arbitrarily small. Certainly in such
> cases spacetime is not "extreme". What do you mean by "sometimes"? Are
> there mathematicians who claim that "spacetime is extreme" at the
> Schwarzschild radius (whatever its value)?

But why on earth do you assume that tidal efects are the arbiter of what
is extreme? It is circular argumentation to use GR to support itself.
If GR breaks down at some point (and most seem to agree nowadays that it
must) then the point where it breaks down is determined not by GR, but
by the underlying, more correct theory.

As an example, if this more correct theory happened to be an effective
field theory of gravitons in flat background spacetime, we would expact
that the breakdown would occur before spacetime became extreme in the
context of the more correct theory. In this case, 'extreme' might
plausibly mean topologically incompatible with a flat background.

(And the breakdown might not manifest in terms of tidal effects
different from those predicted by GR.)

- Gerry Quinn

Igor Khavkine

unread,
Apr 30, 2009, 2:51:42 PM4/30/09
to
On Apr 30, 12:40 pm, Gerry Quinn <ger...@indigo.ie> wrote:
> In article <gt56ju$rn...@online.de>, hel...@astro.multiCLOTHESvax.de
> says...
>
> > In article <nJKFHZFw2X9JF...@charlesfrancis.wanadoo.co.uk>, Oh No

> > <N...@charlesfrancis.wanadoo.co.uk> writes:
>
> > > In practice mathematicians sometimes claim that actually spacetime is
> > > not singular, or even particularly extreme at the Schwarzschild radius.
> > > Nonetheless, I think Dirac's argument is very sound.
>

> > If you make a black hole arbitrarily massive, then the tidal effects etc
> > at the Schwarzschild radius become arbitrarily small. Certainly in such
> > cases spacetime is not "extreme". What do you mean by "sometimes"? Are
> > there mathematicians who claim that "spacetime is extreme" at the
> > Schwarzschild radius (whatever its value)?
>
> But why on earth do you assume that tidal efects are the arbiter of what
> is extreme?

Because there is no evidence to assume otherwise.

> It is circular argumentation to use GR to support itself.

GR is not used to support itself. GR predicts that in the vicinity of
the horizon of a large black hole, the space-time can be highly
accurately approximated by a small deviation from the space-time of
special-relativity. We spend our entire lives in other regions of
space time which are small deviations from that of special relativity.
It is the success of special relativity + small corrections to
describe the physics that we see daily around us that lends
credibility to the assumption that nothing strange happens in the
vicinity of the horizon of a large black hole. In fact, the property
of a region space-time that would distinguish it from a small
deviation from flatness (general relativity) is precisely the presence
of large tidal forces.

> If GR breaks down at some point (and most seem to agree nowadays that it
> must) then the point where it breaks down is determined not by GR, but
> by the underlying, more correct theory.

True, however you may have noticed that we have no information about
what the underlying theory might be. Until more information is
available, we are left with GR to tell us its own limits. In the same
way that the description of waves on the surface of water breaks down
when the gradient of the height of the water level diverges (the wave
breaks), GR itself tells us that it might not be adequate to describe
space-time regions where tidal effects diverge (which is, again, not
at the horizon of a black hole).

> As an example, if this more correct theory happened to be an effective
> field theory of gravitons in flat background spacetime, we would expact
> that the breakdown would occur before spacetime became extreme in the
> context of the more correct theory. In this case, 'extreme' might
> plausibly mean topologically incompatible with a flat background.

Topology is a global property of space-time, which cannot influence
local physics. Even if we are dealing with an effective theory of
gravitons on a flat background, the only internal restriction on its
effectiveness is the presence of large tidal forces.

Igor

Eric Gisse

unread,
Apr 30, 2009, 2:51:41 PM4/30/09
to
On Apr 29, 2:21 am, juanREM...@canonicalscience.com wrote:
> Eric Gisse wrote on Tue, 28 Apr 2009 18:15:07 +0200:
>
> (...)
>
> > On the other hand, the Schwarzschild radius wasn't shown to be an
> > artifact until Kruskal discovered the coordinate chart that smoothly
> > connected the inner and outer regions of a black hole.
>
> "Schwarzschild radius" is not an artifact, just a radius.
>
> The Kruskal Szekres coordinates show that spacetime metric is not
> singular at the "Schwarzschild radius", just that.

I thought it was clear from context that I was referring to the fact
that the metric in Schwarzschild's isotropic coordinates was singular
at the Schwarzschild radius, and that [apparently previous] work had
shown that the singular-ness was an _artifact_ - a coordinate
singularity - as opposed to a true singularity.

>
> Of course, the Kruskal Szekres coordinates continues containing the event
> horizon corresponding to the "Schwarzschild radius".
>
> (...)
>

> --http://www.canonicalscience.org/
>
> Usenet Guidelines:http://www.canonicalscience.org/en/miscellaneouszone/guidelines.html

Gerry Quinn

unread,
May 1, 2009, 5:11:26 AM5/1/09
to
In article <0120544b-c3bc-4212-a441-a612d3b8d5e5
@w31g2000prd.googlegroups.com>, igo...@gmail.com says...

> On Apr 30, 12:40 pm, Gerry Quinn <ger...@indigo.ie> wrote:

> > But why on earth do you assume that tidal efects are the arbiter of what
> > is extreme?
>
> Because there is no evidence to assume otherwise.

Clearly anyone who has doubts about the accuracy of GR at or near the
horizon believes, correctly or otherwise, that there is indeed some
evidence for at least suspecting otherwise. So really the above
argument will convince only those who do not need convincing.



> > It is circular argumentation to use GR to support itself.
>
> GR is not used to support itself. GR predicts that in the vicinity of
> the horizon of a large black hole, the space-time can be highly
> accurately approximated by a small deviation from the space-time of
> special-relativity. We spend our entire lives in other regions of
> space time which are small deviations from that of special relativity.
> It is the success of special relativity + small corrections to
> describe the physics that we see daily around us that lends
> credibility to the assumption that nothing strange happens in the
> vicinity of the horizon of a large black hole.

But doesn't this assumption also depend on the belief that GR both
correctly and fully predicts the nature of spacetime in the vicinity of
the horizon?

> > If GR breaks down at some point (and most seem to agree nowadays that it
> > must) then the point where it breaks down is determined not by GR, but
> > by the underlying, more correct theory.
>
> True, however you may have noticed that we have no information about
> what the underlying theory might be. Until more information is
> available, we are left with GR to tell us its own limits. In the same
> way that the description of waves on the surface of water breaks down

> when the gradient of the height of the water level diverges (>the wave


> breaks), GR itself tells us that it might not be adequate to describe
> space-time regions where tidal effects diverge (which is, again, not
> at the horizon of a black hole).

And yet if the water happens to be enclosed in a box with a lid, the
wave theory will suddenly break down when the waves become high enough
to encounter the lid. And the wave theory will give absolutely no
inkling of when that is about to happen!

Perhaps a lid seems an arbitrary constraint. But a similar problem
exists if the water is of finite depth. When the waves become large
enough that this depth becomes relevant, the wave theory again breaks
down, even though experiments on low energy waves might give no
indications that any such breakdown could occur.

All it takes to nullify the argument that we have no information about
what the underlying theory might be, is to propose a reasonable
alternative theory. For example, as I suggested before, how about an
effective field theory of gravitons on a flat background? What does
this theory - it is incomplete, obviously, but all theories are
incomplete - predict about the limits of GR? The same as GR, or
something different?

GR is just one theory - and indeed a theory not without major problems,
as it seems to be in fundamental disagreement with quantum theory.
Whatever disadvantages gravitons - or at least effective gravitons - may
have, at least they are immune from this problem.

> > As an example, if this more correct theory happened to be an effective
> > field theory of gravitons in flat background spacetime, we would expact
> > that the breakdown would occur before spacetime became extreme in the
> > context of the more correct theory. In this case, 'extreme' might
> > plausibly mean topologically incompatible with a flat background.
>
> Topology is a global property of space-time, which cannot influence
> local physics. Even if we are dealing with an effective theory of
> gravitons on a flat background, the only internal restriction on its
> effectiveness is the presence of large tidal forces.

What I was implying was, in essence, that the predictions of GR for
regions inside the Schwarzschild radius are predictions that seem
clearly incompatible with the predictions of any theory that depends on
a flat background. Thus one would expect that the predictions of any
such theory must necessarily diverge from GR near the horizon.

Remember that the crucial point about the graviton theory is that it is
an *effective* theory; that is to say a low-energy approximation. Thus
it must have an underlying high-energy structure - and it may not matter
much for some purposes what that underlying structure is - which
maintains an effective flat background and creates gravitons and other
particles which operate on it.

To properly discuss the predictions of an effective field theory, the
above must be taken into account. Then it's really not hard to come up
with scenarios in which effects related to the underlying structure
become manifest in regions near the Schwarzschild radius, i.e. regions
which are 'extreme' in the sense I used above.

- Gerry Quinn

CarlBrannen

unread,
May 1, 2009, 2:17:15 PM5/1/09
to
On Apr 30, 3:40�am, Gerry Quinn <ger...@indigo.ie> wrote:
> As an example, if this more correct theory happened to be an effective
> field theory of gravitons in flat background spacetime, we would expact
> that the breakdown would occur before spacetime became extreme in the
> context of the more correct theory. �In this case, 'extreme' might
> plausibly mean topologically incompatible with a flat background.

I agree with this exactly. As it turns out, there is a version of GR
that is based on a flat background (flat in the sense of Minkowski).
It was found by the Cambridge Geometry group, which specializes
in geometric algebra. In the mainstream, geometric algebra can
be thought of as Dirac's gamma matrices, but from a geometric
point of view, without any particular matrix representation.
As a result, their work is used to compute wave functions for
fermions near black holes. A recent intro is here:
http://www.mrao.cam.ac.uk/~anthony/oxford_undergrad_physics_conf_april_2008.pdf
Publications to 2005 are:
http://www.mrao.cam.ac.uk/~clifford/publications/index.html

For a vanilla (Schwarzschild) black hole, the resulting coordinates
are
Gullstrand-Painleve. These coordinates are like Schwarzschild except
that each event in space time ends up with a modified coordinate
time. As a consequence, particles falling into a black hole reach
the singularity at the origin (in finite time) instead of getting
stuck
on the event horizon. I wrote up a Java applet simulation that shows
this and compares Newton's gravity with Schwarzschild and GP
coordinates:
http://www.gravitysimulation.com/

If one works in a fixed flat background, one naturally assumes
that the force of gravity is proportional to the flux of gravitons.
Given a choice of coordinates, one obtains force as a function
of distance, and from this one can compute a correction to the
1/r^2 law that a flux of non interacting gravitons would produce.

This works easiest if one computes the force on a stationary
test particle (stationary in the sense of the choice of coordinates
of course), because then the speed of gravitons cancels out and
one gets a law for how the flux interacts with itself to make more
flux (so as to violate the 1/r^2 law).

I did this calculation and found that the change in flux is
proportional
to the square of the flux density in both Schwarzschild and GP
coordinates. I wrote this up and submitted it to the gravitation essay
contest. A copy is here:
http://www.brannenworks.com/gravity2009.pdf

Another thing included in the above are the exact equations of motion
for test particles in Schwarzschild and GP coordinates. That is,
I removed the extra differential equation from the 4 geodesic
equations to get 3 differential equations as in Newton's case.
That way you can compare Einstein with Newton on a term by
term basis and simulate to see which terms are necessary to
meet the various solar system tests.

Carl

Juan R. González-Álvarez

unread,
May 1, 2009, 10:11:44 PM5/1/09
to
Gerry Quinn wrote on Fri, 01 May 2009 09:11:26 +0000:

(...)

> What I was implying was, in essence, that the predictions of GR for
> regions inside the Schwarzschild radius are predictions that seem
> clearly incompatible with the predictions of any theory that depends on
> a flat background. Thus one would expect that the predictions of any
> such theory must necessarily diverge from GR near the horizon.

Yes, non-metrical theories of gravity do not predict horizons neither BHs.
Details may depend on the specific theory picked out, but a field theory
of gravity predicts observable distributions of matter inside Sch. radius,
intrinsic magnetic fields in condensed objects, redshifted surfaces, extra
radiation mechanism, low luminosity, etc.

So far as I know no proof of existence of BHs and horizons have been given
(indeed more rigorous people continues using the term "candidate to BH"
when discussing certain astrophysical objects), thus the issue is open.

--

Tom Roberts

unread,
May 1, 2009, 10:11:56 PM5/1/09
to
Gerry Quinn wrote:
> What I was implying was, in essence, that the predictions of GR for
> regions inside the Schwarzschild radius are predictions that seem
> clearly incompatible with the predictions of any theory that depends on
> a flat background. Thus one would expect that the predictions of any
> such theory must necessarily diverge from GR near the horizon.

Not really. In GR, spacetime is modeled as a manifold, which means that
surrounding each and every point there is some finite region whose
points can be put in 1-to-1 correspondence to the points of a flat
manifold (of the same dimension). If this other theory is a local theory
on a manifold (i.e. if it has your "flat background [manifold]"), or if
it is expressed in the usual sort of differential equations, then your
expectation is not met.

As someone else pointed out, global topology is irrelevant.
"All physics is local." [attribution lost]

This is why GR is not expected to break down near the horizon of a (big)
black hole. This does not apply "at a singularity", as the size of the
region for that 1-to-1 correspondence goes to zero very rapidly as one
approaches the singularity -- nobody seriously expects GR to be valid there.

In some putative deeper theory in which the model of a manifold breaks
down at the Planck scale, there is no obvious reason to suppose that
models like GR break down until some typical length characterizing the
structure of the manifold approaches the Planck scale [#]. For any black
hole of an earth mass or above, that is far inside the horizon.

[#] Say, within a factor of a billion or so. This is still
a billion times smaller than the charge radius of a proton.


Tom Roberts

Igor Khavkine

unread,
May 1, 2009, 10:12:49 PM5/1/09
to
On May 1, 11:11 am, Gerry Quinn <ger...@indigo.ie> wrote:
> In article <0120544b-c3bc-4212...@w31g2000prd.googlegroups.com>, igor...@gmail.com says...

> > On Apr 30, 12:40 pm, Gerry Quinn <ger...@indigo.ie> wrote:
> > > But why on earth do you assume that tidal efects are the arbiter of what
> > > is extreme?
>
> > Because there is no evidence to assume otherwise.
>
> Clearly anyone who has doubts about the accuracy of GR at or near the
> horizon believes, correctly or otherwise, that there is indeed some
> evidence for at least suspecting otherwise.  So really the above
> argument will convince only those who do not need convincing.

If there is good evidence it should be widely known, yet it
isn't. Perhaps such evidence exists and is of high quality. Then, to
prove this line of reasoning wrong, all you have to do is present it.
Until such time, this reasoning is perfectly convincing.

> > > It is circular argumentation to use GR to support itself.
>
> > GR is not used to support itself. GR predicts that in the vicinity of
> > the horizon of a large black hole, the space-time can be highly
> > accurately approximated by a small deviation from the space-time of
> > special-relativity. We spend our entire lives in other regions of
> > space time which are small deviations from that of special relativity.
> > It is the success of special relativity + small corrections to
> > describe the physics that we see daily around us that lends
> > credibility to the assumption that nothing strange happens in the
> > vicinity of the horizon of a large black hole.
>
> But doesn't this assumption also depend on the belief that GR both
> correctly and fully predicts the nature of spacetime in the vicinity of
> the horizon?  

GR predicts the "nature of spacetime", as you put it; that's just part
of the theory. Whether it does so correctly is not a theoretical
question, but an experimental one. I presume your question aimed to
make this assumption seem unreasonable. On the contrary, consistency
with experiment to date is precisely what makes reasonable the
assumption that GR continues to hold beyond the currently known
limits.

> > > If GR breaks down at some point (and most seem to agree nowadays that it
> > > must) then the point where it breaks down is determined not by GR, but
> > > by the underlying, more correct theory.
>
> > True, however you may have noticed that we have no information about
> > what the underlying theory might be. Until more information is
> > available, we are left with GR to tell us its own limits. In the same
> > way that the description of waves on the surface of water breaks down
> > when the gradient of the height of the water level diverges (>the wave
> > breaks), GR itself tells us that it might not be adequate to describe
> > space-time regions where tidal effects diverge (which is, again, not
> > at the horizon of a black hole).
>
> And yet if the water happens to be enclosed in a box with a lid, the
> wave theory will suddenly break down when the waves become high enough
> to encounter the lid.  And the wave theory will give absolutely no
> inkling of when that is about to happen!  
>
> Perhaps a lid seems an arbitrary constraint.  But a similar problem
> exists if the water is of finite depth.  When the waves become large
> enough that this depth becomes relevant, the wave theory again breaks
> down, even though experiments on low energy waves might give no
> indications that any such breakdown could occur.

You've illustrated your point very well, and I've agreed with it:
extra information (such as the size of the box containing the
water) imposes limits on the validity of the theoretical description
independent of internal limits of the theory.

> All it takes to nullify the argument that we have no information about
> what the underlying theory might be, is to propose a reasonable
> alternative theory.

No, that doesn't nullify anything. Going back to the wave analogy, the
box containing the water can be very large and the water can be very
deep, such that the external restrictions on the applicability of the
theoretical model of surface waves are far weaker than the internal
restrictions due to wave breaking. You proposed a small box, I
proposed a large box. Who is to say which alternative is more
reasonable until we have some information about the existence and size
of a box. No information means no information: you can't use the mere
existence of alternatives to predict plausible limitations of a
theory. In the case of GR, there is no information about external
restrictions on its validity, only internal ones.

> For example, as I suggested before, how about an
> effective field theory of gravitons on a flat background?  What does
> this theory - it is incomplete, obviously, but all theories are
> incomplete - predict about the limits of GR?  The same as GR, or
> something different?

Incidentally, the same as GR, including the prediction that nothing
strange happens at the horizon. There are strong restrictions on what
an effective theory of gravitons might look like if it is extended
beyond linear order. Theorems proven by Weinberg and others
essentially restrict the possibilities to GR + (higher order curvature
corections), IIRC.

> GR is just one theory - and indeed a theory not without major problems,
> as it seems to be in fundamental disagreement with quantum theory.
> Whatever disadvantages gravitons - or at least effective gravitons - may
> have, at least they are immune from this problem.

They happen to not be immune. Pretty much all the problems with
quantization of gravity appear already on the perturbative level.

> > > As an example, if this more correct theory happened to be an effective
> > > field theory of gravitons in flat background spacetime, we would expact
> > > that the breakdown would occur before spacetime became extreme in the
> > > context of the more correct theory.  In this case, 'extreme' might
> > > plausibly mean topologically incompatible with a flat background.
>
> > Topology is a global property of space-time, which cannot influence
> > local physics. Even if we are dealing with an effective theory of
> > gravitons on a flat background, the only internal restriction on its
> > effectiveness is the presence of large tidal forces.
>
> What I was implying was, in essence, that the predictions of GR for
> regions inside the Schwarzschild radius are predictions that seem
> clearly incompatible with the predictions of any theory that depends on
> a flat background.  Thus one would expect that the predictions of any
> such theory must necessarily diverge from GR near the horizon.

Unfortunately for your argument, this is not even true. For a
sufficiently large black hole, the exterior and parts of the interior
black hole space-time (un until where tidal effects become strong) can
be reproduced by the same theory of effective gravitons that you were
proposing earlier. Divergence from GR only takes place in regions of
large curvature (= large tidal effects).

> Remember that the crucial point about the graviton theory is that it is
> an *effective* theory; that is to say a low-energy approximation.  Thus
> it must have an underlying high-energy structure - and it may not matter
> much for some purposes what that underlying structure is - which
> maintains an effective flat background and creates gravitons and other
> particles which operate on it.  
>
> To properly discuss the predictions of an effective field theory, the
> above must be taken into account.  Then it's really not hard to come up
> with scenarios in which effects related to the underlying structure
> become manifest in regions near the Schwarzschild radius, i.e. regions
> which are 'extreme' in the sense I used above.

I think you'll find it harder than you think to come up with such
scenarios. Already, in an effective theory of gravitons the
Schwarzschild radius is in no way extreme.

Igor

Igor Khavkine

unread,
May 1, 2009, 10:13:04 PM5/1/09
to
On May 1, 8:17 pm, CarlBrannen <c...@brannenworks.com> wrote:
> On Apr 30, 3:40 am, Gerry Quinn <ger...@indigo.ie> wrote:
>
> > As an example, if this more correct theory happened to be an effective
> > field theory of gravitons in flat background spacetime, we would expact
> > that the breakdown would occur before spacetime became extreme in the
> > context of the more correct theory. In this case, 'extreme' might
> > plausibly mean topologically incompatible with a flat background.
>
> I agree with this exactly. As it turns out, there is a version of GR
> that is based on a flat background (flat in the sense of Minkowski).
> It was found by the Cambridge Geometry group, which specializes
> in geometric algebra. In the mainstream, geometric algebra can
> be thought of as Dirac's gamma matrices, but from a geometric
> point of view, without any particular matrix representation.
> As a result, their work is used to compute wave functions for
> fermions near black holes. A recent intro is here:
> http://www.mrao.cam.ac.uk/~anthony/oxford_undergrad_physics_conf_april_2008.pdf
> Publications to 2005 are:
> http://www.mrao.cam.ac.uk/~clifford/publications/index.html

Sorry, couldn't find a description of such a "version of GR" in either
of those links. Without more details it's hard to tell whether your
agreement with Gerry Quinn has a solid basis.

> For a vanilla (Schwarzschild) black hole, the resulting coordinates
> are
> Gullstrand-Painleve. These coordinates are like Schwarzschild except
> that each event in space time ends up with a modified coordinate
> time. As a consequence, particles falling into a black hole reach
> the singularity at the origin (in finite time) instead of getting
> stuck
> on the event horizon. I wrote up a Java applet simulation that shows
> this and compares Newton's gravity with Schwarzschild and GP
> coordinates: http://www.gravitysimulation.com/

You're talking about different coordinate systems/patches covering
different parts of the Schwarzschild geometry. No matter how these
different different coordinate systems are obtained, the space-time
geometry is the same. GP coordinates are regular at the horizon, while
Schwarzschild coordinates are not; that's not disputed. However,
nothing gets stuck at the horizon. Any geodesic crossing the horizon
does so in finite *proper* time (a coordinate independent statement).

Igor

Phillip Helbig---remove CLOTHES to reply

unread,
May 2, 2009, 3:49:42 AM5/2/09
to
In article <MPG.24645f3...@news.indigo.ie>, Gerry Quinn
<ger...@indigo.ie> writes:

> In article <0120544b-c3bc-4212-a441-a612d3b8d5e5
> @w31g2000prd.googlegroups.com>, igo...@gmail.com says...
> > On Apr 30, 12:40 pm, Gerry Quinn <ger...@indigo.ie> wrote:
>
> > > But why on earth do you assume that tidal efects are the arbiter of what
> > > is extreme?
> >
> > Because there is no evidence to assume otherwise.
>
> Clearly anyone who has doubts about the accuracy of GR at or near the
> horizon believes, correctly or otherwise, that there is indeed some
> evidence for at least suspecting otherwise.

What is that evidence?

Juan R.

unread,
May 2, 2009, 11:52:21 AM5/2/09
to
CarlBrannen wrote on Fri, 01 May 2009 18:17:15 +0000:

(...)

> I agree with this exactly. As it turns out, there is a version of GR
> that is based on a flat background (flat in the sense of Minkowski). It
> was found by the Cambridge Geometry group, which specializes in
> geometric algebra.

There is *not* another version of GR. GR is an unique theory.

As explained by Wald, Norbert Straumann, and other general relativists,
flat backgrounds have not physical meaning in GR. I like Straumann
comparison of flat backgrounds in GR with "a kind of unobservable ether".

> In the mainstream, geometric algebra can be thought of as Dirac's gamma
> matrices, but from a geometric point of view, without any particular
> matrix representation. As a result, their work is used to compute wave
> functions for fermions near black holes. A recent intro is here:
> http://www.mrao.cam.ac.uk/~anthony/
> oxford_undergrad_physics_conf_april_2008.pdf

I have taked a look to this buy there is no details to know they are
really doing. They give certain geometric treatment for flat spacetime,
but is not known if their GTG is only a mathematical reformulation of GR
or a different (non-metric) theory. The pdf looks misleading at several
parts.

(...)

> If one works in a fixed flat background, one naturally assumes that the
> force of gravity is proportional to the flux of gravitons. Given a
> choice of coordinates, one obtains force as a function of distance, and
> from this one can compute a correction to the 1/r^2 law that a flux of
> non interacting gravitons would produce.

If you are working within the framework of a non-metric theory, then
gravity is explained as a force over flat background (the force in a first
approximation may be explained by a graviton model).

If you are working within GR, then above is nonsensical. There is *not*
gravitational forces in GR. Motion is geodesic. This is well explained in
Wald textbook.

(...)

Regards

Bob_for_short

unread,
May 3, 2009, 4:46:36 AM5/3/09
to
> Going back to the wave analogy, the
> box containing the water can be very large and the water can be very
> deep, such that the external restrictions on the applicability of the
> theoretical model of surface waves are far weaker than the internal
> restrictions due to wave breaking. You proposed a small box, I
> proposed a large box. Who is to say which alternative is more
> reasonable until we have some information about the existence and size
> of a box. No information means no information: you can't use the mere
> existence of alternatives to predict plausible limitations of a
> theory. In the case of GR, there is no information about external
> restrictions on its validity, only internal ones.

A box with water is a too complicated example. I propose to look at
the ideal gas state equation:

PV=NkT. Obviously it does not contain restrictions of its
applicability due to phase transitions.

Another example: F=-kX. It does not contain limits on its
applicability at big X.

Classical Electrodynamics with its electric charge radius r0=e^2/
(mc^2) ceases to be valid well before r0.

Existing the horizon is already a big problem of a theory, in my
opinion, somewhat similar to CED's one.
Apparently it is the "geometrization" of interaction, together with
the self-action, who are responsible for
non physical solutions.

Bob.

CarlBrannen

unread,
May 3, 2009, 5:37:05 AM5/3/09
to
On May 1, 7:13�pm, Igor Khavkine <igor...@gmail.com> wrote:
> Sorry, couldn't find a description of such a "version of GR" in either
> of those links. Without more details it's hard to tell whether your
> agreement with Gerry Quinn has a solid basis.

There are a lot of geometric algebra papers listed besides the
ones applying to gravity. The primary GR paper is:

A. N. Lasenby, C. J. L. Doran and S. F. Gull.
Gravity, gauge theories and geometric algebra
Phil. Trans. R. Soc. Lond. A356, 487-582 (1998).
http://www.mrao.cam.ac.uk/~clifford/publications/abstracts/gravity.html

It's also on arXiv and has citebase entry:
http://www.citebase.org/abstract?id=oai%3AarXiv.org%3Agr-qc%2F0405033

Sometimes it helps to have the same theory explained by a different
writer. David Hestenes' papers on this version of GR. His website is:
http://modelingnts.la.asu.edu/

He has three papers discussing the foundations and extending the
theory:

Spacetime Calculus for Gravitation Theory
"a systematic account of the mathematical formalism"
http://modelingnts.la.asu.edu/pdf/NEW_GRAVITY.pdf

Gauge Theory Gravity with Geometric Calculus
"A new unitary formulation of Einstein's tensor leads to
resolution of long-standing problems with energy momentum
conservation in general relativity."
http://modelingnts.la.asu.edu/pdf/GTG.w.GC.FP.pdf

Spacetime Geometry with Geometric Calculus
Geometric Calculus is developed for curved-space treatments of General
Relativity and comparison with the flat-space gauge theory approach by
Lasenby, Doran and Gull. Einstein's Principle of Equivalence is
generalized
to a gauge principle that provides the foundation for a new
formulation of
General Relativity as a Gauge Theory of Gravity on a curved spacetime
manifold. Geometric Calculus provides mathematical tools that
streamline
the formulation and simplify calculations. The formalism automatically
includes spinors so the Dirac equation is incorporated in a
geometrically
natural way.
http://modelingnts.la.asu.edu/pdf/SpacetimeGeometry.w.GC.proc.pdf

By "version of GR", what I mean is that the theory gives results
identical to those of GR for situations that can be covered by a
single chart. One loses things like wormholes and that sort of
sci-fi physics. I see that as an improvement over GR.

Of related interest:

Bound States and Decay Times of Fermions in a Schwarzschild Black Hole
Background
http://www.mrao.cam.ac.uk/~clifford/publications/abstracts/bound_states.html

Geometric Algebra, Dirac Wavefunctions and Black Holes
http://www.mrao.cam.ac.uk/~clifford/publications/abstracts/anl_erice_2001.html

Also see mention of GP coordinates here:

This is an elegant generalization of GP coordinates to Kerr metric
(rotating
black holes):
http://www.mrao.cam.ac.uk/~clifford/publications/abstracts/newkerr.html

Fermion absorption cross section of a Schwarzschild black hole
Chris Doran, Anthony Lasenby, Sam Dolan and Ian Hinder
We study the absorption of massive spin-half particles by a small
Schwarzschild black hole by numerically solving the single-particle
Dirac equation in Painleve-Gullstrand coordinates.
Phys.Rev. D 71, 124020 (2005)
"An advantage of our choice of metric is that the Dirac equation can
now be written in a manifestly Hamiltonian form"
http://www.mrao.cam.ac.uk/~clifford/publications/abstracts/bh_absorb.html


Eric Gisse

unread,
May 3, 2009, 10:51:07 AM5/3/09
to
On May 1, 11:49�pm, hel...@astro.multiCLOTHESvax.de (Phillip Helbig---
remove CLOTHES to reply) wrote:
> In article <MPG.24645f39a3e140b3...@news.indigo.ie>, Gerry Quinn

>
> <ger...@indigo.ie> writes:
> > In article <0120544b-c3bc-4212-a441-a612d3b8d5e5
> > @w31g2000prd.googlegroups.com>, igor...@gmail.com says...

> > > On Apr 30, 12:40 pm, Gerry Quinn <ger...@indigo.ie> wrote:
>
> > > > But why on earth do you assume that tidal efects are the arbiter of what
> > > > is extreme?
>
> > > Because there is no evidence to assume otherwise.
>
> > Clearly anyone who has doubts about the accuracy of GR at or near the
> > horizon believes, correctly or otherwise, that there is indeed some
> > evidence for at least suspecting otherwise. �
>
> What is that evidence?

Since you have ruled out an expenditure of personal effort by asking
the question...

S. Doeleman, et. al : "Event horizon-scale structure in the
supermassive black hole candidate at the galactic center", Nature 455,
4 Sept. 2008, p78,
Ghez, et. al : "Stellar orbits around the galactic center black hole",
ApJ 2-05; 744-757 [arXiv:astro-ph/0306130]

Eric Gisse

unread,
May 3, 2009, 11:15:59 AM5/3/09
to
On May 1, 6:11�pm, "Juan R. Gonz�lez-�lvarez"

<email.addr...@not.given> wrote:
> Gerry Quinn wrote on Fri, 01 May 2009 09:11:26 +0000:
>
> (...)
>
> > What I was implying was, in essence, that the predictions of GR for
> > regions inside the Schwarzschild radius are predictions that seem
> > clearly incompatible with the predictions of any theory that depends on
> > a flat background. �Thus one would expect that the predictions of any
> > such theory must necessarily diverge from GR near the horizon.
>
> Yes, non-metrical theories of gravity do not predict horizons neither BHs.

Is that a fact?

Given that the dynamics of Sgr A* and the associated 20 or so nearby
[> ~50AU, < parsec] stars have been well studied for a decade now
[Ghez, et. al - frequently in ApJ], this claim should be easy to test.
Especially with VLBI imagery of the current black hole at Sgr A*.

Is it that all of them "do not predict" or is it that nobody has
looked? A review paper or something approximating a published
reference would be nice as opposed to making the claim and not backing
it up.

> Details may depend on the specific theory picked out, but a field theory
> of gravity predicts observable distributions of matter inside Sch. radius,

Given that infalling matter onto a black hole has been observed
without any visible display of impact (as opposed to impact upon a
neutron star), any theory that predicts this will need to some fancy
tapdancing.

> intrinsic magnetic fields in condensed objects, redshifted surfaces, extra
> radiation mechanism, low luminosity, etc.

All of which have no observational support.

>
> So far as I know no proof of existence of BHs and horizons have been given
> (indeed more rigorous people continues using the term "candidate to BH"
> when discussing certain astrophysical objects), thus the issue is open.

Some "rigorous" people will never be satisfied no matter how heavy the
burden of proof is. Those "rigorous" people are a stalwart of
sci.physics and sci.physics.relativity.

S. Doeleman, et. al "Event horizon-scale structure in the supermassive
black hole candidate at the galactic center", Nature 455, 4 Sept.

2008, p 78.

VLBI imagery constrains the object to be about the size of the
Schwarzschild radius of an object of the expected mass from Ghez's
work given respective [~50%] uncertainties & complications from
lensing.

Worst case, the object's mass is within a few Schwarzschild radii. One
can't help but wonder what would be accepted as evidence given that a
black hole, by its' very nature, does not lend itself to immediate
observation. If nothing comes out (in terms of light or other atomic
spew) when a star falls in, or that four million solar masses is
stuffed inside an area less than that of Pluto's orbit with no visible
signature, what is it? Maybe its' a unicorn.

>
> --http://www.canonicalscience.org/
>
> Usenet Guidelines:http://www.canonicalscience.org/en/miscellaneouszone/guidelines.html

jacques

unread,
May 4, 2009, 5:52:43 AM5/4/09
to
On 27 avr, 07:46, Melroy <melroysoa...@hotmail.com> wrote:
> Hi,
> From watching this talkhttp://online.kitp.ucsb.edu/online/joint98/mottola/

Einstein in 1939 claimed that the collapse of a ball of dust under
the Schwarzshild radius was physicaly impossible because, while
collapsing, dust orbiting at the boundary of the ball should reach the
velocity of light before hitting the Schwarzschild radius. The demo
was correct, but the conclusion is wrong. Einstein sould have
concluded that a "static" solution was not possible which is true, but
a dynamic solution is not excluded.
As soon as 1916 Schwarzschild in a paper "Uber das Gravitationsfeld
einer kugel aus inkompressibler Flussigkeit nach der Einsteinchen
Theorie" also claimed that a ball of a incompresible fluid would not
collapsed under the Schwarzschild radius as the pressure at the center
would become infinite before reaching the Schwarzschild radius.

In fact non singular solutions (on the horizon ) were found as soon as
1921 (Painlev�"Compte rendu � l'academie des sciences") and well
understood in 1932 by Lema�tre "L'univers en expansion" who derived a
non singular solution (on the horizon) directly from the Einstein
equation, using generic sphericaly symetric coordinates, understanding
and explaining that the trick of singulatity on the horizon is that
people would get a static solution as the physical solution is not
static but in fact stationnary (in the modern assumption).

Describing the Black hole phenomenology is far more simple in these
coordinates than in Schwarzschild coordinates as the time coordinate
is the proper time of a radial free falling observer, in fact comoving
with space collapse.

Obviously the singularity at the center remains in all cases and we a
may have doubt on the validity of GR at this point and closely around.

But for the horizon, there is no physical singularity at all, even
tidal effect can be very small in case of very large black holes. This
does not mean that the horizon is not physical, it is in fact, but GR
looks perfectly suitable for describing most physical BH's.

Juan R. González-Álvarez

unread,
May 4, 2009, 11:09:05 PM5/4/09
to
CarlBrannen wrote on Sun, 03 May 2009 11:37:05 +0200:

(...)

> A. N. Lasenby, C. J. L. Doran and S. F. Gull. Gravity, gauge theories
> and geometric algebra Phil. Trans. R. Soc. Lond. A356, 487-582 (1998).
> http://www.mrao.cam.ac.uk/~clifford/publications/abstracts/ gravity.html

Here the authors claim his GTG to be not GR and "crucial question to
address is whether any experimental tests are likely to distinguish
between general relativity and GTG in the immediate future."

Experiments and observations addressed to differentiate GR from non-
metric theories: measurement of scalar radiation in binary pulsars,
measurement of intrinsic magnetic fields in ROCs, fractal cosmological
observations, spin in Galileo experiment, Newtonian limits, etc. There was
a discussion about this in last PPC-08 international conference

http://www.canonicalscience.org/en/publicationzone/
canonicalsciencetoday/20080516.html

More discussion about experimental and observational status of GR and
alternative (non-metric) theories as nonlinear relativistic field theory,
Eugene V. Stefanovich relativistic theory of gravity, Weber & Mach
relational theory by Andre K. T. Assis & Peter Graneau, Yuriy Sergeyevich
Vladimirov's relativistic relational theory, and Stuckelberg, Horwitz, &
Piron action-at-a-distance theory worked by Matthew A. Trump & William C.
Schieve is given in

http://www.canonicalscience.org/en/publicationzone/drafts.html

This also includes a proof that only three of above theories predict the
correct cosmological boundaries. As Joy Christian have rightfully
remarked, the boundaries used in GR are observational falsified [#]:

However, physical evidence clearly suggests that we are not living in an
'island universe' (cf. Penrose 1996, 593-594) i.e., universe is not 'an
island of matter surrounded by emptiness'.

> Spacetime Calculus for Gravitation Theory "a systematic account of the
> mathematical formalism"
> http://modelingnts.la.asu.edu/pdf/NEW_GRAVITY.pdf

This reference is not so good as above. Here the author start citing
Poincare work on a theory of gravity using flat spacetime. I agree that
Poincare solution is "the more advantageous", but Poincare did not work
GR. Poincare worked a field theory of gravity. Hestenes adds:

"Applied to GR, this amounts to the claim that any curved-space
formulation of the theory can be replaced by an equivalent and simpler
flat-space formulation. Ironically, the curved-space formulation has
been preferred by nearly everyone since the inception of GR."

The first part is not correct. And there is not irony in the second. The
reason which the curved-space formulation of GR has been preferred in
virtually any treatment is because as explained by Wald, Straumann, and
others, the flat spacetime background has not physical meaning in GR. As
Straumann remarks, and I agree, the flat background represents "a kind of
unobservable ether" in GR.

Unfortunately, Hestenes seems to confound GR (a metric theory) with
Poincare work in FTG (a non-metric theory).

This paper looks misleading and confusing at several parts. E.g. he uses a
metric, but in (7.7) is not known if he is working the GR metric or the
metric of a gauge theory.

He also claims that the metric tensor in GR is physically interpreted as a
gravitational potential. This is not true. The tensor g_ab is not a
gravitational potential, unless one want consider that in absence of
gravity \eta_ab was a gravitational potential!

If he means that h_ab for g_ab = \eta_ab + h_ab is a gravitational
potential, again the response is not. The fact that h_ab cannot be
associated to a gravitational field is reason that gravity cannot be
interpreted in terms of forces in GR.

(7.11) seems to confirm that he confound the spacetime metric g_ab in
metric theories as GR with the effective metric {g_ab}_eff in gauge
theories as field theory of gravity. both gs are very similar but not the
same object. The mathematics is different.

> Gauge Theory Gravity with Geometric Calculus "A new unitary formulation
> of Einstein's tensor leads to resolution of long-standing problems with
> energy momentum conservation in general relativity."
> http://modelingnts.la.asu.edu/pdf/GTG.w.GC.FP.pdf

Idem. Moreover, could someone explain to me how unitary transformations on
GR converts energy momentum pseudotensors in tensors?

(...)

Regards


[#]
Why the Quantum Must Yield to Gravity 1999: arXiv:GR-qc/9810078v3.
Christian, Joy.


--

Gerry Quinn

unread,
May 4, 2009, 11:16:20 PM5/4/09
to
In article <0c5246df-128c-4a33-acce-3f89bd43fe86
@c9g2000yqm.googlegroups.com>, igo...@gmail.com says...
> On May 1, 11:11 am, Gerry Quinn <ger...@indigo.ie> wrote:

> > For example, as I suggested before, how about an
> > effective field theory of gravitons on a flat background?  What does


> > this theory - it is incomplete, obviously, but all theories are

> > incomplete - predict about the limits of GR?  The same as GR, or


> > something different?
>
> Incidentally, the same as GR, including the prediction that nothing
> strange happens at the horizon. There are strong restrictions on what
> an effective theory of gravitons might look like if it is extended
> beyond linear order. Theorems proven by Weinberg and others
> essentially restrict the possibilities to GR + (higher order curvature
> corections), IIRC.

Okay, this is kind of getting to the central issue of my argument. A
whole lot of points were raised in other posts, and I'll try to answer
some of them elsewhere without creating a swarm of separate posts, but
the core of what I am interested in lies here.

I do understand there are some potential issues with the sorts of
gravitons we can hypothesise, e.g. the Weinberg-Witten theorem might
mean composite gravitons would have to be massive or non Lorentz-
invariant. Maybe there are more issues too, and maybe I underestimate
these issues. But most theories in any controversial field of physics
have some issues somewhere.

What I was supposing, however - is it wrong? - is that one can
nevertheless write down a theory of effective gravitons operating on a
flat Minkowski background, with some energy cutoff above which another
unknown theory is assumed to operate. And that whatever theoretical
issues may arise, they do not immediately rule out the possibility of
making such a theory work.

Now if we have any decent theory of physics, what we can do - at least
in principle - is use it to model parts of the universe, given
appropriate boundary conditions. Let's say we do that with the above
theory. We pick boundary conditions describing a cubic region of space
containing enough matter to collapse under its own gravitation. We
model this system using our graviton theory and see how it evolves.

Since the graviton part of this theory does not contain any explicit
mention of spacetime geometry, any modelling can in principle be done
without any reference to coordinate changes or geometry of any kind
beyond that of the flat Minkowski background. Any valid solution given
by our theory can therefore be written down in such terms.

Now it seems to me that any such solution must diverge from the GR
Schwarzschild solution near the black hole horizon. Because the
SChwarzschild solution contains an interior region that cannot be
modelled in terms of the flat Minkowski spacetime of the graviton
solution. This is what I was referring to when I mentioned topology; a
model of the universe with non-simple topology can't be the same as one
with simple topology. In the case of the effective graviton theory as
described above - if such a theory exists and is workable - there is
after all a whole universe of particles above the energy cutoff which
might for all we know be completely unaffected by gravity, so there is
no point arguing that a non-simple topology can in any way evolve from
the theory.

Does you agree with the above? If so, I guess what you are saying is
that there is no workable theory of effective gravitons in flat space.
And yet that seems very strange, unless the whole concept of gravitons
is some bizarre mathematical trap!

I don't really 'get' the concept of assuming a curved spacetime, and
working out the consequences of gravitons operating on that. Sure, I
can see why you might do it sometimes if it makes the math easier, or
helps confirm that you get similar answers to GR in some particular
regime. But in the end, if you are assuming a curved spacetime that
isn't ultimately just a geometric model of interactions between
gravitons on a flat background, what's the point of building a graviton
theory at all?

> > What I was implying was, in essence, that the predictions of GR for
> > regions inside the Schwarzschild radius are predictions that seem
> > clearly incompatible with the predictions of any theory that depends on

> > a flat background.  Thus one would expect that the predictions of any


> > such theory must necessarily diverge from GR near the horizon.
>
> Unfortunately for your argument, this is not even true. For a
> sufficiently large black hole, the exterior and parts of the interior
> black hole space-time (un until where tidal effects become strong) can
> be reproduced by the same theory of effective gravitons that you were
> proposing earlier. Divergence from GR only takes place in regions of
> large curvature (= large tidal effects).

Am I correct in assuming that the above only applies when you divorce
the graviton theory from its flat background and apply the graviton part
of it on spacetimes of arbitrary curvature, where the initial generation
of this curvature by gravitons is not modelled?

If so, the above says nothing to my argument, because the possibility
exists that a part of the complete theory - i.e. the flat background -
has been discarded at some point in the analysis.

If you take the graviton part alone, it's unsurprising that you would
end up with GR. But in fact we know that a pure graviton theory doesn't
work. To my mind that's telling us that the other part of the theory -
the imperfection that means only an effective theory can be valid - is
just as important when it comes to describing the world.

- Gerry Quinn

Gerry Quinn

unread,
May 4, 2009, 11:17:09 PM5/4/09
to
In article <qZDKl.15547$hc1....@flpi150.ffdc.sbc.com>, tjroberts137
@sbcglobal.net says...

> Gerry Quinn wrote:
> > What I was implying was, in essence, that the predictions of GR for
> > regions inside the Schwarzschild radius are predictions that seem
> > clearly incompatible with the predictions of any theory that depends on
> > a flat background. Thus one would expect that the predictions of any
> > such theory must necessarily diverge from GR near the horizon.
>
> Not really. In GR, spacetime is modeled as a manifold, which means that
> surrounding each and every point there is some finite region whose
> points can be put in 1-to-1 correspondence to the points of a flat
> manifold (of the same dimension). If this other theory is a local theory
> on a manifold (i.e. if it has your "flat background [manifold]"), or if
> it is expressed in the usual sort of differential equations, then your
> expectation is not met.
>
> As someone else pointed out, global topology is irrelevant.
> "All physics is local." [attribution lost]
>
> This is why GR is not expected to break down near the horizon of a (big)
> black hole. This does not apply "at a singularity", as the size of the
> region for that 1-to-1 correspondence goes to zero very rapidly as one
> approaches the singularity -- nobody seriously expects GR to be valid there.

I think you're missing the point I was making which does speak only to
the global nature of topology. In a nutshell, I think that if two
theories give solutions with very different global topology, then these
solutions must be different, and thus the two theories must be
intrisically different.

The two topologies in question are the Schwarzschild solution, and flat
Minkowski space. If they could be transformed into each other after
cutting out the central singularity of the Schwarzschild solution, there
wouldn't really be a problem either. But I don't think this is enough
to allow the transformation! As far as I can see the transformation
becomes impossible at the outermost trapped surface, i.e. the event
horizon. Thus, I conclude that the solutions much diverge before this.
(Obviously the event horizon is not the same for all observers, but that
is unimportant in this context, as a breakdown in GR need not be
abrupt.)

As to whether this is a problem for gravitons or GR, that is a matter
for debate, and I am probably in the minority here in thinking it's a
problem for GR. But the substantive issue I'm raising here is whether
it is a fatal problem for at least one of them. Many people seem to
think they can happily coexist in the context of black hole interiors,
and I'm arguing that that makes no sense.

- Gerry Quinn

Jonathan Thornburg [remove -animal to reply]

unread,
May 5, 2009, 1:59:02 AM5/5/09
to
Melroy <melroy...@hotmail.com> wrote:
> From watching this talk http://online.kitp.ucsb.edu/online/joint98/mottola/

> it seems that Einstein and Dirac has strong issues against the
> existence of black holes
> and had published stuff in 1939 and 1962 respectively arguing against
> black holes.
> But this was before the work by Penrose in 60s.
> Einstein's 1939 paper is published in Ann. of Math 40, 922
> Can some experts (Jonathan Thornburg, Steve Carlip etc) point to me
> the mistake in Einstein's 1939 paper and how
> the current understanding of the picture on gravitational collapse
> circumvents Einstein's arguments in his 1939 paper. I have not seen
> much discussion of these papers on arxiv or in this newsgroup.

By any reasonable measure of "deep understanding of analytical
calculations in general relativity" I am surely at least an order
of magnitude less knowledgable than Steve Carlip. But having said
that, the mistake is (from today's perspective) both elementary
and quite instructive to contemplate, so I'll plunge ahead:

If you're interested in black holes, it's absolutely essential
to realise that the basic concept of (what we now call) a black
hole was not widely understood until the 1960s. A few researchers
realised this earlier, but you should still look with extreme
skepticism on any paper written about (what we now call) black
holes before the mid-1960s.
[I believe the phrase "black hole" was first coined
by Wheeler sometime around 1969. Previously the phrase
"frozen star" was sometimes used. Since Schwarzschild
discovered his solution to the Einstein equations in
1916, the term "Schwarzschild solution" has been widely
used ever since then, and remains common.]
And yes, that skepticism should include papers by Einstein.

When thinking about general relativity (GR), it's often useful to
consider the analogy of latitude and longitude as coordinates on the
Earth's surface. These are perfectly good coordinates on most of the
Earth's surface... but if you try to use them in a neighborhood of
the north or south pole, you'll get into trouble, because these
coordinates are singular there. (This shows up, for example, as
the derivative

d (longitude)
---------------------------------------
d (locally measured distance in meters)

blowing up at the pole. In differential-geometry language, this
derivative is a component of the inverse metric.) Continuing the
analogy, this does *not* mean that the Earth's surface has a weird
shape at the pole. It just means we've chosen an inappropriate set
of coordinates for doing navigation there.

What we see here is that, just because our coordinates are singluar,
there's not necessarily anything wrong with physics. More generally,
<emphasis>
coordinates have no intrinsic physical meaning!
</emphasis>
This latter statement carries over perfectly into GR, and failing
to heed it is the underlying cause of many of the incorrect articles
about GR that litter the internet.

In GR, if we write the Schwarzschild metric in (among others)
isotropic coordinates, where the line element is
[N.b. I'm using the usual GR units where G=c=1.]

ds^2 = -A dt^2 + B (dx^2 + dy^2 + dz^2)

where A = ((1 - M/2r)/(1 + M/2r))^2
B = (1 + M/2r)^4

then it's clear that A, the time-time metric coefficient, vanishes
at r=2m, and hence the inverse metric blos up there. But as we've
seen, this tells us *nothing* at all about whether spacetime is or
isn't singular there. In this case, the non-singularity of spacetime
can be proven fairly easily:
* One way is to explicitly compute the components of the Riemann
tensor in the orthonormal frame of an observer who falls into
the black hole. This calculation is done in MTW section 31.2.
[N.b. "MTW" = Misner, Thorne, & Wheeler, "Gravitation",
W. H. Freeman 1973, paperback ISBN 0-7167-0344-0]
* Another way is to exhibit a different coordinate system such
that the same (Schwarzschild) spacetime has a manifestly
non-singular metric written in our new coordinates.
I'm not going to write these down here, because they're
readily available: MTW section 31.4 gives several such
non-singular coordinate systems, including Eddington-Finkelstein
coordinates
http://en.wikipedia.org/wiki/Eddington-Finkelstein_coordinates
and Kruskal-Szekeres coordinates
http://en.wikipedia.org/wiki/Kruskal-Szekeres_coordinates
This last Wikipedia article is very nice, and has diagrams which
greatly aid in understanding what's going on.

The non-singular nature of Schwarzschild spacetime at r=2m is
particularly apparent when the metric is written in Kruskal-Szekeres
coordinates,
ds^2 = (32M^3/r) exp(-r/2M) (-dT^2 + dR^2)
+ r^2 (dr^2 + sin^2(theta) dtheta^2)
where it's quite obvious that nothing "funny" happens at r=2M.
See the Wikipedia article I cited above for details.

Thus, we see that r=2M is a singularity of the isotropic coordinates,
but not a singularity of Schwarzschild spacetime.

With this background, let's look at the first page of Einstein's 1939
paper, which may be found online at
http://www.jstor.org/pss/1968902

Einstein writes:

it is noted that
(equation for the time-time metric component)
vanishes for r = mu/2. [Einstein uses a Greek "mu" for the mass.]
This means that a clock kept at this place would go at the rate zero.

From today's perspective this is true, although it's rather less
surprising if one observes that the locally-measured acceleration
needed to keep a clock at this place is infinite (!).

Einstein then writes

Further it is easy to show that both light rays and material
particles take an infintely long time (measured in "coordinate time")
in order to reach the point r = mu/2 when originating from a point
r > mu/2.

Again, this is perfectly correct -- Einstein has quite explicitly said
that he's considering *coordinate* time.

Einstein then writes

In this sense the sphere r = mu/2 constitutes a place where the
field is singular.

Now we see the problem: Einstein is confusing the singularity of
various *coordinate* measurements at r=2M (e.g., the *coordinate*
time to fall in to r=2M from outside), with the singularity of
the [gravitational] "field", i.e., of spacetime itself. This was
(is) the key mistake in this paper: It does indeed take an infinite
interval of isotropic-coordinate time to fall in to r=2M from outside.
But if we use a more suitable time coordinate (e.g., proper time
of the falling observer), it takes a finite time. It's not r=2M
that's the problem, it's the coordinate choice!

To repeat: There's *nothing* "funny" or odd about spacetime at
or near r=2M in Schwarzschild spacetime. You can't send signals
from inside r=2M to outside r=2M, but that's a *global* property
of the causal structure -- there's nothing a local observer can
detect which is "funny" at r=2M.

[Indeed, the inability to send signals to certain spacetime
regions is already present in *special* relativity: I'll describe
that in a following posting.]

There's a larger lesson to be learned from Einstein's mistake,
namely that science is cumulative. Thus scientific problems which
were hard at some time in the past may be easy today. Einstein made
this mistake 70 years ago. If he were alive today, I'm sure he
would not make this mistake. Indeed, I would expect anyone getting
a grade of B or better in an undergraduate beginning-GR course today
to not make this mistake. Hopefully, in the year 2079, many of
the scientific problems which challenge us today will be similarly
well understood.

--
-- "Jonathan Thornburg [remove -animal to reply]" <jth...@astro.indiana-zebra.edu>
Dept of Astronomy, Indiana University, Bloomington, Indiana, USA
"Washing one's hands of the conflict between the powerful and the
powerless means to side with the powerful, not to be neutral."
-- quote by Freire / poster by Oxfam

Eric Gisse

unread,
May 5, 2009, 1:59:02 AM5/5/09
to
On May 4, 7:09 pm, "Juan R. Gonz�lez-�lvarez"
<email.addr...@not.given> wrote:

[...]

> More discussion about experimental and observational status of GR and
> alternative (non-metric) theories as nonlinear relativistic field theory,
> Eugene V. Stefanovich relativistic theory of gravity, Weber & Mach
> relational theory by Andre K. T. Assis & Peter Graneau, Yuriy Sergeyevich
> Vladimirov's relativistic relational theory, and Stuckelberg, Horwitz, &
> Piron action-at-a-distance theory worked by Matthew A. Trump & William C.
> Schieve is given in
>
> http://www.canonicalscience.org/en/publicationzone/drafts.html
>
> This also includes a proof that only three of above theories predict the
> correct cosmological boundaries. As Joy Christian have rightfully
> remarked, the boundaries used in GR are observational falsified [#]:

Why is a draft that is unavailable for public consumption continually
cited?

It is great that you feel you have a litany of proofs written but when
nobody can see them, nobody can critically analyze what was written.
When is that expected to change?

For example, you assert that binary pulsar observations indicate the
presence of scalar gravitational radiation. Except there's no support
for the claim, eg: http://www.sciencemag.org/cgi/content/full/304/5670/547?ck=nck
and references therein. In fact, I see no real adherent to the claims
except by Baryshev which goes back to the early 80's. Not that there
was support then, either - the error bars were simply larger.

[...]

> [#]
> Why the Quantum Must Yield to Gravity 1999: arXiv:GR-qc/9810078v3.
> Christian, Joy.

This is not an observational falsification.

Knowing that GR is wrong at some limit is not the same thing as having
an _observation_ of the incorrectness of GR at that limit.

> --http://www.canonicalscience.org/
>
> Usenet Guidelines:http://www.canonicalscience.org/en/miscellaneouszone/gu=
idelines.html

juanR...@canonicalscience.com

unread,
May 5, 2009, 1:59:53 AM5/5/09
to
Eric Gisse wrote on Sun, 03 May 2009 15:15:59 +0000:

(...)

>> So far as I know no proof of existence of BHs and horizons have been
>> given (indeed more rigorous people continues using the term "candidate
>> to BH" when discussing certain astrophysical objects), thus the issue
>> is open.
>
> Some "rigorous" people will never be satisfied no matter how heavy the
> burden of proof is. Those "rigorous" people are a stalwart of
> sci.physics and sci.physics.relativity.
>
> S. Doeleman, et. al "Event horizon-scale structure in the supermassive
> black hole candidate at the galactic center", Nature 455, 4 Sept. 2008,
> p 78.

Precisely this is my point. First you would notice their use of "black
hole candidate" in their title, just as I propose. Now take a look to the
abstract

http://www.nature.com/nature/journal/v455/n7209/abs/nature07245.html

The cores of most galaxies *are thought* to harbour supermassive black
holes [...] Sgr A* [...] is the closest example of this phenomenon, with
an *estimated black hole mass* that is 4,000,000 times that of the Sun
[...] This is less than the *expected* apparent size of the event horizon
of the *presumed* black hole [...]

I like their *precise* use of the terms: are thought, estimated, expected=
,
presumed... This contrast with your usual exagerations and personal
dreams.

Moreover a comprensehive reading of his work shows they did not prove
existence of black holes.

(...)

--

Juan R.

unread,
May 6, 2009, 3:54:07 AM5/6/09
to
Eric Gisse wrote on Tue, 05 May 2009 05:59:02 +0000:

(...)

> For example, you assert that binary pulsar observations indicate the
> presence of scalar gravitational radiation.

The main part of my message to Carl was about the GTG theory, but you
deleted all including the references.

Your incorrect statements about my work already were recently replied in
sci.physics.foundations moderated group. E.g. here

http://groups.google.com/group/sci.physics.foundations/msg/c41f906658213e6f

or here

http://groups.google.com/group/sci.physics.foundations/msg/bf1dcdb7d65dc3d1

I have also given you adittional references and data and I see no reason
to repeat all that again.

(...)

> In fact, I see no real adherent to the claims except by Baryshev which
> goes back to the early 80's.

Baryshev works I have revised are at least about 20 years more modern than
you say. You also deleted the link to the PPC-08 Conference in your
response.

(...)

Eric Gisse

unread,
May 6, 2009, 5:42:13 AM5/6/09
to
On May 5, 11:54�pm, "Juan R." Gonz�lez-�lvarez

<juanREM...@canonicalscience.com> wrote:
> Eric Gisse wrote on Tue, 05 May 2009 05:59:02 +0000:
>
> (...)
>
> > For example, you assert that binary pulsar observations indicate the
> > presence of scalar gravitational radiation.
>
> The main part of my message to Carl was about the GTG theory, but you
> deleted all including the references.

...to stuff I don't have any interest in? I suppose you are right.

You did, however, make some claims I _am_ interested in. They may not
occupy the vast majority of your message but I do consider brevity to
be a virtue on occasion.

>
> Your incorrect statements about my work already were recently replied in
> sci.physics.foundations moderated group. E.g. here
>
> http://groups.google.com/group/sci.physics.foundations/msg/c41f906658213e6f

"Moreover, there is discussion of PPN formalism and PPN parameters of
geometric (re)formulations of alternative theories are given in the
report."

"Your cite of Will preprint is also unuseful. His last update of his
paper in Living reviews Relativity is one of references used in my
work."

"The topic of strong gravity is also addressed in the report."

You are technically right in that you replied to my statements. They
just aren't useful because as I have stated previously, your work _is
not public_.

>
> or here
>
> http://groups.google.com/group/sci.physics.foundations/msg/bf1dcdb7d65dc3d1

"It is ironic that you cite above reference by Taylor and Weisberg,
when it is the reference number *17* in my draft."

"The primary source for my claims is not one USENET old message in a
noisy non-moderated group."

"Moreover, their recent work is basic for my discussion, of the
uncertainties in binary pulsar parameters and the link with the recent
claims by other authors of possible detection of scalar waves. "

...etc

Since I'm apparently being quite opaque in my request, I'll be a
little more clear.

This is following a pattern that has been repeated several times in
the past few years. I'm growing tired of it.

Your draft is a horrible primary source. Nobody but you has access to
it, and given that you are unwilling or unable to answer simple
questions about your claims of observational falsifications of general
relativity re: cosmological boundary conditions (the remark was
snipped without comment) or scalar gravitational radiation.

>
> I have also given you adittional references and data and I see no reason
> to repeat all that again.

Your references are all the same: your draft!

I see many claims to works by other authors but you never actually
bother to take the few seconds to paste the literature references. For
example, you claim there is a credible detection of scalar
gravitational radiation. Do you have a reference for that which _is
not_ your draft? Do you have a reference from beyond the mid 1990's?

>
> (...)
>
> > In fact, I see no real adherent to the claims except by Baryshev which
> > goes back to the early 80's.
>
> Baryshev works I have revised are at least about 20 years more modern than
> you say.

When I say "goes back to the early 80's", it does not mean that it was
the only time. I see about 15 years of him making the claim, except
now that it is 2009 the claim is a lot harder to back up given the
tightness of the error bars. It would also be instructive to
understand that the largest source of error in the observations of
B1913+16 are from uncertainties associated with ignorance of the Earth-
pulsar distance.

http://www.astro.spbu.ru/staff/baryshev/c_v.htm

Furthermore, his most recent work on the topic was over a decade ago.
1980 + 20 is larger than 1997. Someone recently mentioned that
exaggerations are bad.

If you have _evidence_ that PSR B1913+16 shows evidence for scalar
gravitational radiation / disagrees with general relativity, then I'd
love to see it. However, I am wholly uninterested in hearing the words
"...in my draft..." because nobody but you has access to it..

> You also deleted the link to the PPC-08 Conference in your
> response.

I left it out for many reasons. Which one would you consider most
suitable?

a) The conference is not relevant to the current discussion.
b) The conference link does not work.
c) The many paragraphs associated with the conference link were and
are completely irrelevant to the claims I am continually trying to get
you to substantiate.

If there is a reason the conference is relevant, I'd be glad to
consider it. Especially if you could provide a working link.

>
> (...)
>
> --http://www.canonicalscience.org/
>
> Usenet Guidelines:http://www.canonicalscience.org/en/miscellaneouszone/guidelines.html

Igor Khavkine

unread,
May 6, 2009, 1:07:42 PM5/6/09
to
On May 5, 5:16�am, Gerry Quinn <ger...@indigo.ie> wrote:
> In article <0c5246df-128c-4a33-acce-3f89bd43fe86
> @c9g2000yqm.googlegroups.com>, igor...@gmail.com says...

> What I was supposing, however - is it wrong? - is that one can
> nevertheless write down a theory of effective gravitons operating on a
> flat Minkowski background, with some energy cutoff above which another
> unknown theory is assumed to operate. �And that whatever theoretical
> issues may arise, they do not immediately rule out the possibility of
> making such a theory work.

Sure, you can have a theory of effective gravitons on a flat
background. I believe I've agreed with statement several times now.
Feel free to assume that this point is well established. The problems
come when you make incorrect deductions about the properties of such a
theory.

> Now if we have any decent theory of physics, what we can do - at least
> in principle - is use it to model parts of the universe, given
> appropriate boundary conditions. �Let's say we do that with the above
> theory. �We pick boundary conditions describing a cubic region of space
> containing enough matter to collapse under its own gravitation. �We
> model this system using our graviton theory and see how it evolves.

That's a fine thing to model. However, to be specific, what dimension
do you have in mind? A 3d cube or a 4d cube?

[regarding solutions of an effective theory:]


> Now it seems to me that any such solution must diverge from the GR
> Schwarzschild solution near the black hole horizon.

It might seem to you that way, but it's just not true.

> Because the
> SChwarzschild solution contains an interior region that cannot be
> modelled in terms of the flat Minkowski spacetime of the graviton
> solution. This is what I was referring to when I mentioned topology; a
> model of the universe with non-simple topology can't be the same as one
> with simple topology.

And your reasoning here is flawed. Now, I'm a little confused by your
use of the term "topology". Are you referring to the fact that an
eternal Schwarzschild black hole has topology of R^2 x S^2 as opposed
to R^4 of Minkowski space? Or are you referring to something else? At
the moment I can't come up with plausible alternatives keeping the
standard meaning of the term "topology". Well, keeping that terms
standard meaning, there is absolutely no relation between the local
properties of the Schwarzschild horizon (which is what at question
here) and the global topological properties of the space-time. You are
simply using a non-sequitur, unless you can explain what you mean by
"topology" and show it's somehow tied to the properties of the
horizon.

> I don't really 'get' the concept of assuming a curved spacetime, and
> working out the consequences of gravitons operating on that. �Sure, I
> can see why you might do it sometimes if it makes the math easier, or
> helps confirm that you get similar answers to GR in some particular
> regime. �But in the end, if you are assuming a curved spacetime that
> isn't ultimately just a geometric model of interactions between
> gravitons on a flat background, what's the point of building a graviton

> theory at all? �

I in turn don't get what you mean here. There is no mystery to
gravitons, on curved or flat backgrounds. Einstein's equations are non-
linear. If you know an exact solution, more solutions can be found by
approximating them with small deviations from the known solution.
These deviations are gravitons. This is no different from
approximating the bottom of any particle potential with a harmonic
oscillator.

> > > What I was implying was, in essence, that the predictions of GR for
> > > regions inside the Schwarzschild radius are predictions that seem
> > > clearly incompatible with the predictions of any theory that depends on

> > > a flat background. Thus one would expect that the predictions of any


> > > such theory must necessarily diverge from GR near the horizon.
>
> > Unfortunately for your argument, this is not even true. For a
> > sufficiently large black hole, the exterior and parts of the interior

> > black hole space-time (up until where tidal effects become strong) can


> > be reproduced by the same theory of effective gravitons that you were
> > proposing earlier. Divergence from GR only takes place in regions of
> > large curvature (= large tidal effects).
>
> Am I correct in assuming that the above only applies when you divorce
> the graviton theory from its flat background and apply the graviton part
> of it on spacetimes of arbitrary curvature, where the initial generation
> of this curvature by gravitons is not modelled?

Umm, say what? I said nothing of the sort, and neither have you,
before this lats post. Let me repeat what I said, in hopefully more
precise language:

For a sufficiently large black hole, all the gravitational effects
predicted by general relativity in a Schwarzschild space-time for
regions of small curvature (which include the exterior, the horizon,
and parts of the interior) can be reproduced to desired accuracy by an
effective spin-2 field theory formulated on a Minkowski background. At
the same time, large curvature regions of the Schwarzschild space-time
cannot be reproduced with a similar spin-2 field theory.

> If so, the above says nothing to my argument, [...]

Well, whatever way you read my comments, it obviously cannot be "so",
since they directly defeat your argument of divergence between GR and
an effective graviton theory at the Schwarzschild horizon. You may be
uncertain as to the truth of what I said, but you can't dismiss it as
irrelevant.

Igor

Eric Gisse

unread,
May 6, 2009, 5:23:34 PM5/6/09
to
On May 4, 9:59 pm, juanREM...@canonicalscience.com wrote:
> Eric Gisse wrote on Sun, 03 May 2009 15:15:59 +0000:
>
> (...)
>
> >> So far as I know no proof of existence of BHs and horizons have been
> >> given (indeed more rigorous people continues using the term "candidate
> >> to BH" when discussing certain astrophysical objects), thus the issue
> >> is open.
>
> > Some "rigorous" people will never be satisfied no matter how heavy the
> > burden of proof is. Those "rigorous" people are a stalwart of
> > sci.physics and sci.physics.relativity.
>
> > S. Doeleman, et. al "Event horizon-scale structure in the supermassive
> > black hole candidate at the galactic center", Nature 455, 4 Sept. 2008,
> > p 78.
>
> Precisely this is my point. First you would notice their use of "black
> hole candidate" in their title, just as I propose. Now take a look to the
> abstract
>
> http://www.nature.com/nature/journal/v455/n7209/abs/nature07245.html

Or even better...read the article! Important things are written
inside. Relevant things. Things you should see before judging.

Perhaps we could start calling it the unicorn candidate. Sure that's
nonsense but like you say, we don't really _know_ now do we?

>
> The cores of most galaxies *are thought* to harbour supermassive black
> holes

...as supported by observation...

> [...] Sgr A* [...] is the closest example of this phenomenon, with
> an *estimated black hole mass* that is 4,000,000 times that of the Sun

"estimated" like the Earth-Sun distance is "estimated" to be 93
million miles.

Ghez, et. al : "Stellar orbits around the galactic center black hole
", ApJ 2-05 ; 744-757

> [...] This is less than the *expected* apparent size of the event horizon


> of the *presumed* black hole [...]

So?

"It is an open question whether or not the Sgr A* source is centred on
the black hole. Indeed, several models predict an offset between Sgr
A* and the black hole position."

Even though I equate Sgr A* with the associated black hole, there's
plenty of reason to doubt that the radio emission source Sgr A* is not
at the exact same position as the black hole.

Read the last 2 paragraphs, as opposed to selective quoting of the
abstract. We don't have a true fix of the inclination of the accretion
disk, and the models used assumed a _not rotating_ black hole even
though there is ample reason to expect that the black hole has some
rotation. The upper bound on the rotation parameter is something like
0.6 as I recall, but anything above 0.1 is going to tweak the results.

>
> I like their *precise* use of the terms: are thought, estimated, expected=

=

Let us not forget that you do not apply this overly rigid standard of
uncertainty to _your_ work, in which you use "proof" way too much
about what chalks up to personal preference given the lack of evidence
otherwise.

> ,
> presumed... This contrast with your usual exagerations and personal
> dreams.

That's not true.

I only had the unicorn for a little while and its' death was entirely
accidental. How was I supposed to know a trillion shots of jager were
too many?

>
> Moreover a comprensehive reading of his work shows they did not prove
> existence of black holes.

...and a comprehensive reading of other works do not "prove" the
existence of neutron stars, extrasolar planets, or prions.

Mostly because science doesn't prove. It does, however, test a given
hypothesis. And the black hole idea is pretty consistent with
observation.

If the comprehensive reading you just mentioned has any evidence
_against_ the existence of black holes it would not only be relevant
but worth mentioning. Is there any such evidence?

>
> (...)
>
> --http://www.canonicalscience.org/
>
> Usenet Guidelines:http://www.canonicalscience.org/en/miscellaneouszone/gu=
idelines.html

Juan R.

unread,
May 7, 2009, 4:40:35 AM5/7/09
to
Eric Gisse wrote on Wed, 06 May 2009 21:23:34 +0000:

(...)

>> > S. Doeleman, et. al "Event horizon-scale structure in the
>> > supermassive black hole candidate at the galactic center", Nature
>> > 455, 4 Sept. 2008, p 78.
>>
>> Precisely this is my point. First you would notice their use of "black
>> hole candidate" in their title, just as I propose. Now take a look to
>> the abstract
>>
>> http://www.nature.com/nature/journal/v455/n7209/abs/nature07245.html

(...)

> Perhaps we could start calling it the unicorn candidate. Sure that's
> nonsense but like you say, we don't really _know_ now do we?

Give me a citation of an author equating the scientific and rigorous term
"black hole candidate" with your ridiculous term "unicorn candidate" or I
will conclude that again you do not understand the difference between
rigorous scientific research and sci-fi movies or between scientific
knowledge and your own misconceptions.

>> The cores of most galaxies *are thought* to harbour supermassive black
>> holes
>
> ...as supported by observation...

says "are thought" not "are observed".

>> [...] Sgr A* [...] is the closest example of this phenomenon, with an
>> *estimated black hole mass* that is 4,000,000 times that of the Sun
>
> "estimated" like the Earth-Sun distance is "estimated" to be 93 million
> miles.

An *estimated black hole mass* of 4 millions (was 3.7 before and 2.6
earlier...) has not the same status than Earth-Sun distance.

I know nobody doubting about the existence of the distance, but Sgr A*
continues being considerd a bright and very compact astronomical radio
source *supposed* to be the location of a supermassive black hole.

> Ghez, et. al : "Stellar orbits around the galactic center black hole",
> ApJ 2-05 ; 744-757

I would prefer a more rigorous "around the pressumed galactic center black
hole".

There is many possibilities and alternatives we cannot rule out.

http://arxiv.org/abs/0902.0346

"In summary, what our calculation suggests is that it might be possible
to have a black hole without having a black hole."

Black hole mimickers: regular versus singular behavior 2008: Phys. Rev.
D78, 024040. José P. S. Lemos, Oleg B. Zaslavskii

How to tell a gravastar from a black hole 2007: Class. Quant. Grav. 24,
4191-4206. Cecilia B. M. H. Chirenti, Luciano Rezzolla

"Gravastars have been recently proposed as potential alternatives to
explain the astrophysical phenomenology traditionally associated to
black holes, raising the question of whether the two objects can be
distinguished at all."

Etc.

>> [...] This is less than the *expected* apparent size of the event
>> horizon of the *presumed* black hole [...]
>
> So?

So I propose to continue using precise terms as "presumed black hole" and
similar ones.

> "It is an open question whether or not the Sgr A* source is centred on
> the black hole. Indeed, several models predict an offset between Sgr A*
> and the black hole position."
>
> Even though I equate Sgr A* with the associated black hole, there's
> plenty of reason to doubt that the radio emission source Sgr A* is not
> at the exact same position as the black hole.
>
> Read the last 2 paragraphs, as opposed to selective quoting of the
> abstract.

Contrary you can believe, from your own misreadings, the authors are not
contradicting their own abstract.

(...)

>> Moreover a comprensehive reading of his work shows they did not prove
>> existence of black holes.
>
> ...and a comprehensive reading of other works do not "prove" the
> existence of neutron stars, extrasolar planets, or prions.
>
> Mostly because science doesn't prove.

Remind above was my reply to your unfounded claim:

Some "rigorous" people will never be satisfied no matter how heavy the
burden of proof is.

Eric Gisse

unread,
May 7, 2009, 6:56:44 AM5/7/09
to
On May 7, 12:40�am, "Juan R." Gonz�lez-�lvarez

<juanREM...@canonicalscience.com> wrote:
> Eric Gisse wrote on Wed, 06 May 2009 21:23:34 +0000:
>
> (...)
>
> >> > S. Doeleman, et. al "Event horizon-scale structure in the
> >> > supermassive black hole candidate at the galactic center", Nature
> >> > 455, 4 Sept. 2008, p 78.
>
> >> Precisely this is my point. First you would notice their use of "black
> >> hole candidate" in their title, just as I propose. Now take a look to
> >> the abstract
>
> >>http://www.nature.com/nature/journal/v455/n7209/abs/nature07245.html
>
> (...)
>
> > Perhaps we could start calling it the unicorn candidate. �Sure that's
> > nonsense but like you say, we don't really _know_ now do we?
>
> Give me a citation of an author equating the scientific and rigorous term
> "black hole candidate" with your ridiculous term "unicorn candidate" or I
> will conclude that again you do not understand the difference between
> rigorous scientific research and sci-fi movies or between scientific
> knowledge and your own misconceptions.

You missed the point.

The object known as the "black hole" has specific predictions, all of
which are satisfied by current observation in the instance of Sgr A*.

But since you seem to think there is still a fair amount of
uncertainty (that you can't quantify), we might as well call it the
unicorn candidate because it doesn't matter what is found you just
won't be satisfied.

>
> >> The cores of most galaxies *are thought* to harbour supermassive black
> >> holes
>
> > ...as supported by observation...
>
> says "are thought" not "are observed".

I'm gonna go with my original thought of "as supported by
observation".

How many documented examples of galaxies with supermassive dark
objects pushing past a million solar masses would you like? I can
think of one in which the spin has been reasonably well measured, and
several others in ApJ that are simply taken for granted.

Just do me a favor and tell me where the goal post is so it can't be
moved when it is reached.

>
> >> [...] Sgr A* [...] is the closest example of this phenomenon, with an
> >> *estimated black hole mass* that is 4,000,000 times that of the Sun
>
> > "estimated" like the Earth-Sun distance is "estimated" to be 93 million
> > miles.
>
> An *estimated black hole mass* of 4 millions (was 3.7 before and 2.6
> earlier...) has not the same status than Earth-Sun distance.

Oh, so because the mass changed because of improved observations it is
just an _estimation_ ?

a) Look at the error bars. For the last few years, the values have
been pretty consistent within margin - around 4 million solar masses.
b) The older value of 2.6 million solar masses used a value for the
Earth-Sgr A* distance that had larger uncertainties than its' current
value, and it was also based on the orbit of exactly *one* star - S2.
http://www.mpe.mpg.de/ir/GC/res_s2orbit.php?lang=en

Regardless, this is all nitpicking. You are trying to wedge in
uncertainty that does not exist by pointing to what appear to be
conflicting results without an understanding of the state of the art
OR how it has progressed in the last decade.

>
> I know nobody doubting about the existence of the distance, but Sgr A*
> continues being considerd a bright and very compact astronomical radio
> source *supposed* to be the location of a supermassive black hole.

We are all open to alternatives that pass the laugh test.

>
> > Ghez, et. al : "Stellar orbits around the galactic center black hole",
> > ApJ 2-05 ; 744-757

Actually it is ApJ 620. Usually I'm pretty good with proper
transcription. That's why I write in pencil!

>
> I would prefer a more rigorous "around the pressumed galactic center black
> hole".

Well I guess reading the title and immediately dismissing it _is_ an
alternative to reading the abstract or even, god forbid, the entire
article itself.

If you have a problem with Ghez's methodology, I'd be glad to hear it.
Otherwise, don't dismiss an article just because it doesn't play to
your prejudices.

>
> There is many possibilities and alternatives we cannot rule out.

We most certainly can make a damn good guess, by a very loose usage of
GR as well!

Buchdhal's theorem states that for a spherically symmetric
distribution of mass with reasonable energy conditions, under
hydrostatic equilibrium, the _MAXIMUM_ amount of mass M you can stuff
inside a given radius R is bounded by 4Rc^2 / 9 G. The orbital
kinematics put a naive upper bound on the size of the object to be
about 45AU - 500 Schwarzschild radii. Doeleman's work moves the bound
to less than a half dozen radii.

There's no way, not with GR.

>
> http://arxiv.org/abs/0902.0346
>
> � "In summary, what our calculation suggests is that it might be possible
> � to have a black hole without having a black hole."

Only if you buy the assumption that vacuum polarization is relevant
during collapse. That's one of many major assumptions that are pretty
flimsy.

>
> Black hole mimickers: regular versus singular behavior 2008: Phys. Rev.

> D78, 024040. José P. S. Lemos, Oleg B. Zaslavskii

I don't have access to recent Phys. Rev. D issues, but since you
dismissed Ghez's work out of hand I don't suppose you'll make a fuss.

>
> How to tell a gravastar from a black hole 2007: Class. Quant. Grav. 24,
> 4191-4206. Cecilia B. M. H. Chirenti, Luciano Rezzolla
>
> � �"Gravastars have been recently proposed as potential alternatives to
> � �explain the astrophysical phenomenology traditionally associated to
> � �black holes, raising the question of whether the two objects can be
> � �distinguished at all."
>
> Etc.

Have you actually _read_ about the object? It is not anything this
side of "acceptable" as an alternative.

Given that you won't (or can't) read Ghez's paper in ApJ, and clicking
on the "Full Text" button regarding Doeleman's work was too hard (or
you don't have access), I'll presume that Class. Quan. Grav. is
outside your reach. So here's an open access description of the
Gravastar - tell me if you think it passes the laugh test.

http://www.fc.up.pt/pessoas/luis.beca/phisky/PhiSky%20Wiltshire.pdf

>
> >> [...] This is less than the *expected* apparent size of the event
> >> horizon of the *presumed* black hole [...]
>
> > So?
>
> So I propose to continue using precise terms as "presumed black hole" and
> similar ones.

Can I expect such equally precise terminology for your personal
preferences regarding field theories of gravitation versus metric
theories?

>
> > "It is an open question whether or not the Sgr A* source is centred on
> > the black hole. Indeed, several models predict an offset between Sgr A*
> > and the black hole position."

That's what the article says. Comment?

>
> > Even though I equate Sgr A* with the associated black hole, there's
> > plenty of reason to doubt that the radio emission source Sgr A* is not
> > at the exact same position as the black hole.

Do you doubt this?

>
> > Read the last 2 paragraphs, as opposed to selective quoting of the
> > abstract.
>
> Contrary you can believe, from your own misreadings, the authors are not
> contradicting their own abstract.

Do you have a specific argument you would like to make?

>
> (...)
>
> >> Moreover a comprensehive reading of his work shows they did not prove
> >> existence of black holes.
>
> > ...and a comprehensive reading of other works do not "prove" the
> > existence of neutron stars, extrasolar planets, or prions.
>
> > Mostly because science doesn't prove.
>
> Remind above was my reply to your unfounded claim:
>
> � Some "rigorous" people will never be satisfied no matter how heavy the
> � burden of proof is.

It has the virtue of being true.

>
> --http://www.canonicalscience.org/
>
> Usenet Guidelines:http://www.canonicalscience.org/en/miscellaneouszone/guidelines.html


juanR...@canonicalscience.com

unread,
May 7, 2009, 7:51:01 AM5/7/09
to
Eric Gisse wrote on Wed, 06 May 2009 09:42:13 +0000:

(...)

>> http://groups.google.com/group/sci.physics.foundations/msg/
c41f906658213e6f

You only cite some fragments from this link. Among the parts you do not
quote in your present message are the corrections to your several
misunderstandings about PPN and metric formalisms. As then I said "You do
not understand PPN formalism."

> "Moreover, there is discussion of PPN formalism and PPN parameters of
> geometric (re)formulations of alternative theories are given in the
> report."

This was my response to your false statement "this is a masive reinvention
of the wheel" and your other misunderstandings about the scientific
method.

(...)

>> http://groups.google.com/group/sci.physics.foundations/msg/
bf1dcdb7d65dc3d1

Again you only cite some fragments from this link. Among the parts you do
not quote in your present message are your failure to provide a quotation
from mine supporting one of your false accusations, my corrections to your
wrong comment about spin-0 waves, your confusion between GR and nonlinear
field theory, your confusion about "scalar couplings to the action"...

> "It is ironic that you cite above reference by Taylor and Weisberg, when
> it is the reference number *17* in my draft."

This was my response to your false statement of day 20.

(...)

>> > In fact, I see no real adherent to the claims except by Baryshev
>> > which goes back to the early 80's.
>>
>> Baryshev works I have revised are at least about 20 years more modern
>> than you say.
>
> When I say "goes back to the early 80's", it does not mean that it was
> the only time. I see about 15 years of him making the claim,

Either 1997 is your new definition for early 80s or you were again
exposed ;-)

> except now
> that it is 2009 the claim is a lot harder to back up given the tightness
> of the error bars.

But it was already proved in above cited links to spf that you do not
understand any reference on binary pulsars, was using GR or using some
alternative theory, was from 1997, 2004, 2006, or from 2025.

(...)

>> You also deleted the link to the PPC-08 Conference in your response.
>
> I left it out for many reasons. Which one would you consider most
> suitable?
>
> a) The conference is not relevant to the current discussion. b) The
> conference link does not work. c) The many paragraphs associated with
> the conference link were and are completely irrelevant to the claims I
> am continually trying to get you to substantiate.

Your a) is another of your typical false statement because there was a
sesion on gravitational observations, which included discussion about
binary pulsars.

Your b) is right. It was a working link. A search in Google by "Problems
of Practical Cosmology 2008 conference" return the same link
"ppc08.astro.spbu.ru/". Google has a chached version of 21 Feb 2009

http://209.85.229.132/search?q=cache:eTKNHCW_40sJ:ppc08.astro.spbu.ru/
+Problems+of+Practical+Cosmology+2008+conference

There is little I can do if is not working now...

c) Idem

--

Gerry Quinn

unread,
May 7, 2009, 7:52:42 AM5/7/09
to
In article <ffcb9040-7222-469f-b3ba-18a64becf220
@o30g2000vbc.googlegroups.com>, igo...@gmail.com says...

> On May 5, 5:16�am, Gerry Quinn <ger...@indigo.ie> wrote:
> > In article <0c5246df-128c-4a33-acce-3f89bd43fe86

> > Now if we have any decent theory of physics, what we can do - at least


> > in principle - is use it to model parts of the universe, given
> > appropriate boundary conditions. �Let's say we do that with the above
> > theory. �We pick boundary conditions describing a cubic region of space
> > containing enough matter to collapse under its own gravitation. �We
> > model this system using our graviton theory and see how it evolves.
>
> That's a fine thing to model. However, to be specific, what dimension
> do you have in mind? A 3d cube or a 4d cube?

Well, I said 'space' leaving the issue of time open... for generality
one could allow for infinite time, which seems to be unnecessary for
space.

[snipped some]

> For a sufficiently large black hole, all the gravitational effects
> predicted by general relativity in a Schwarzschild space-time for
> regions of small curvature (which include the exterior, the horizon,
> and parts of the interior) can be reproduced to desired accuracy by an
> effective spin-2 field theory formulated on a Minkowski background.

This is exactly what I was asking! I guess our difficulty in
communicating was due to the fact that I have been finding the above
concept problematical (particularly the bit about 'parts of the
interior'), whereas it obviously seems natural to you.

> At
> the same time, large curvature regions of the Schwarzschild space-time
> cannot be reproduced with a similar spin-2 field theory.

I just separated this point from the above as I don't have any problems
with it.

> > If so, the above says nothing to my argument, [...]
>
> Well, whatever way you read my comments, it obviously cannot be "so",
> since they directly defeat your argument of divergence between GR and
> an effective graviton theory at the Schwarzschild horizon. You may be
> uncertain as to the truth of what I said, but you can't dismiss it as
> irrelevant.

And indeed I don't! It is of course 100% relevant as it's exactly the
issue I was, wrongly or rightly, getting at. As to the truth of it,
I've been trying hard to visualise it as described by you. I did find a
very nice web page which morphs various black hole spacetime diagrams
<http://casa.colorado.edu/~ajsh/schwp.html#finkelstein>. And I was
almost convinced, by the Eddington-Finkelstein diagrams, and the free
fall diagrams.

An issue still remains for me, though. Suppose we imagine that in our
cube of simulated space, there is a grid of observers spread throughout
it. Let us say they are made of whatever stuff exists at a higher
energy level than the graviton cutoff, and they communicate with each
other using high energy particles that travel at c and are completely
unaffected by gravity. They watch whatever is going on as the black
hole forms and thereafter, including experiments with infalling
astronauts and whatnot, but they don't get involved.

Let us say these observers draw a diagram of the things they see,
including the trajectories of free-falling particles or light rays, with
horizontal lines representing the observers' mutual understanding of
time. Would this be the same as the Schwarzschild diagram? Or is there
some other named diagram that describes the trajectories of light and
matter as seen by such observers?

What do the paths of infalling light rays in the interior region look
like to them in their diagram? Are they moving forward in time, and
what speed do they go? (We assume their high energy particles travel at
c. At a long distance from the black hole our high-energy observers
will be in agreement with ordinary observers. Closer to the horizon,
but outside it, they will see ordinary light travelling slower than c.
Inside it...?)

Another typical question: our grid of observers watch a black-hole
jumping astronaut who starts falling from a point distant from the black
hole at a certain time. They watch him fall until he gets close to the
singularity (which we will ignore for now). The astronaut's watch says
he has fallen for one hour. Our observers will assess the time as being
somewhat longer. On what diagram can I read off exactly how long they
assess it as?

- Gerry Quinn


Eric Gisse

unread,
May 7, 2009, 11:47:36 AM5/7/09
to
On May 7, 3:51�am, juanREM...@canonicalscience.com wrote:

[...pointless...]

> >> > In fact, I see no real adherent to the claims except by Baryshev
> >> > which goes back to the early 80's.
>
> >> Baryshev works I have revised are at least about 20 years more modern
> >> than you say.
>
> > When I say "goes back to the early 80's", it does not mean that it was
> > the only time. I see about 15 years of him making the claim,
>
> Either 1997 is your new definition for early 80s or you were again
> exposed ;-)

http://www.springerlink.com/content/m68m213384480u54/

This article was also listed on Baryshev's C.V., which apparently you
didn't even look at.

[....]

> >> You also deleted the link to the PPC-08 Conference in your response.
>
> > I left it out for many reasons. Which one would you consider most
> > suitable?
>
> > a) The conference is not relevant to the current discussion. b) The
> > conference link does not work. c) The many paragraphs associated with
> > the conference link were and are completely irrelevant to the claims I
> > am continually trying to get you to substantiate.
>
> Your a) is another of your typical false statement because there was a
> sesion on gravitational observations, which included discussion about
> binary pulsars.

Where might that discussion be located?

http://ppc08.astro.spbu.ru/text_proc.html

[site is working today, as opposed to before]

It isn't any part of the listed parts of the conference. Searching the
site directly gives an .rtf by Taganov

http://ppc08.astro.spbu.ru/materials/thesis/50.rtf

but the assumptions are quite silly.

When the best argument put forth is interpreting the error bar as an
"excess", then I am wholly unimpressed. I see more wishing and hoping
than anything even close to an actual observation.

This is covering no new or interesting grounds. This is simply more of
the same repetition of the claim without support. If you have no
actual literature reference I won't respond to any followup.

Juan R. González-Álvarez

unread,
May 7, 2009, 10:33:41 PM5/7/09
to
Eric Gisse wrote on Thu, 07 May 2009 17:47:36 +0200:

> On May 7, 3:51��� am, juanREM...@canonicalscience.com wrote:

(...)

> http://ppc08.astro.spbu.ru/text_proc.html
>
> [site is working today, as opposed to before]
>
> It isn't any part of the listed parts of the conference. Searching the
> site directly gives an .rtf by Taganov
>
> http://ppc08.astro.spbu.ru/materials/thesis/50.rtf
>
> but the assumptions are quite silly.
>
> When the best argument put forth is interpreting the error bar as an
> "excess", then I am wholly unimpressed. I see more wishing and hoping
> than anything even close to an actual observation.

First you said was not relevant information about pulsars, after you said
could not find the information, now you claim to find a .rtf by "Taganov",
but is not...

The important part is that .rtf proves that your argument against scalar
radiation

http://groups.google.com/group/sci.physics.foundations/msg/
bf1dcdb7d65dc3d1

"Given that the pulsar orbits are decaying within 0.13 +/- 0.21 %
according to Taylor, it sort-of puts the claim into a little bit of
doubt."

was completely without basis. You just do not understand that available
observations of the binary pulsar are compatible both with Weisberg &
Taylor analysis up to 0.2% error and with Baryshev analysis up to 0.1%.

You also avoid to cite other references and observations, and
of course, you also omit to cite other interesting works presented at
PPC-08 about scalar radiation.

Eric Gisse

unread,
May 8, 2009, 12:16:29 PM5/8/09
to
On May 7, 6:33�pm, "Juan R. Gonz�lez-�lvarez"
<email.addr...@not.given> wrote:
[...]

> The important part is that .rtf proves that your argument against scalar
> radiation
>
> http://groups.google.com/group/sci.physics.foundations/msg/
> bf1dcdb7d65dc3d1
>
> � "Given that the pulsar orbits are decaying within 0.13 +/- 0.21 %
> � according to Taylor, it sort-of puts the claim into a little bit of
> � doubt."
>
> was completely without basis.

Uhhh, it is completely _with_ basis.

>From the paper pulled off the conference site: "This is interestingly
close to the expected value 0.735 % for the additional energy loss
predicted for scalar gravitational radiation"

Being off by a factor of 5 is interesting, just not helpful. Perhaps
you have a more relevant reference?

> You just do not understand that available
> observations of the binary pulsar are compatible both with Weisberg &
> Taylor analysis up to 0.2% error and with Baryshev analysis up to 0.1%.

Let's see Baryshev's analysis. Got a reference / link / direction?

>
> You also avoid to cite other references and observations, and
> of course, you also omit to cite other interesting works presented at
> PPC-08 about scalar radiation.

Mostly because they are well hidden. No mention of such things are
anywhere present on the PPC site, and the google site search revealed
exactly what I showed.

It is your claim, you need to back it up.

>
> --http://www.canonicalscience.org/
>
> Usenet Guidelines:http://www.canonicalscience.org/en/miscellaneouszone/guidelines.html

Juan R. González-Álvarez

unread,
May 8, 2009, 7:35:27 PM5/8/09
to
Igor Khavkine wrote on Wed, 06 May 2009 19:07:42 +0200:

> On May 5, 5:16 am, Gerry Quinn <ger...@indigo.ie> wrote:
>> In article <0c5246df-128c-4a33-acce-3f89bd43fe86
>> @c9g2000yqm.googlegroups.com>, igor...@gmail.com says...

(...)

> For a sufficiently large black hole, all the gravitational effects
> predicted by general relativity in a Schwarzschild space-time for
> regions of small curvature (which include the exterior, the horizon,
> and parts of the interior) can be reproduced to desired accuracy by
> an effective spin-2 field theory formulated on a Minkowski background.
> At the same time, large curvature regions of the Schwarzschild space-

> time cannot be reproduced with a similar spin-2 field theory.

If you do a matematical reformulation of GR over flat spacetime, then
it is not suprising you obtain the same results than GR.

If you write down a theory of physical field over flat spacetime, then
you do not obtain neither horizon nor central singularity due to the
energy of the graviton field (energy which is not well defined in GR)

This is why Quinn (as others) wait observable desvitations from GR
predictions.

--

Gerry Quinn

unread,
May 9, 2009, 10:26:35 AM5/9/09
to
In article <pan.2009.05...@canonicalscience.com>,
email....@not.given says...

I'll just say here that I speak only for myself, and that neither the
energy of the graviton field, nor the question (debated on another
subthread) of whether certain astronomical objects are observably
different from GR black holes, loom large in my thinking.

In fact, when it comes to the latter subject I think that maximally
condensed objects probably look much the same as GR black holes, because
while they would (if I am correct) be vastly different close to the
Schwarzschild radius, the signals coming from this region would be so
greatlyly red-shifted that the differences would be hard for a distant
observer to distinguish.

Thus my arguments are more concerned with consistency; the consistency
of GR with *all* parts of a theory of effective gravitons in flat
spacetime - including the high energy sector - is the issue I am
addressing here.

- Gerry Quinn

Juan R.

unread,
May 10, 2009, 8:35:59 AM5/10/09
to
Eric Gisse wrote on Thu, 07 May 2009 12:56:44 +0200:

> On May 7, 12:40�am, "Juan R." Gonz�lez-�lvarez
> <juanREM...@canonicalscience.com> wrote:
>> Eric Gisse wrote on Wed, 06 May 2009 21:23:34 +0000:

(...)

>> Black hole mimickers: regular versus singular behavior 2008: Phys.
>> Rev. D78, 024040. Jose P. S. Lemos, Oleg B. Zaslavskii


>
> I don't have access to recent Phys. Rev. D issues, but since you
> dismissed Ghez's work out of hand I don't suppose you'll make a fuss.

You have no access, but you dismiss the reference!



>> How to tell a gravastar from a black hole 2007: Class. Quant. Grav.
>> 24, 4191-4206. Cecilia B. M. H. Chirenti, Luciano Rezzolla
>>
>> � �"Gravastars have been recently proposed as potential alternatives
>> � �to explain the astrophysical phenomenology traditionally
>> � �associated to black holes, raising the question of whether the
>> two�objects can be distinguished at all."
>>
>> Etc.
>
> Have you actually _read_ about the object? It is not anything this
> side of "acceptable" as an alternative.

This is another comment without content. It is particularly funny, when
one notices that Luciano Rezzolla is a known expert in black holes.

Are you claiming that a known expert in black holes do not understand
black holes?

Are you claiming that referees of Class. Quant. Grav. accept papers
as this without reading them?

http://www.iop.org/EJ/search?query1=gravastar&searchfield2=header_text
%2Ctitle&submit=1

Latter paper in gravastars is from this same year

http://www.iop.org/EJ/abstract/-search=63278752.1/0264-9381/26/2/025003

You would sustain claims as yours. Let us know *your* analysis of
Chirenti and Rezzolla work.


--

juanR...@canonicalscience.com

unread,
May 10, 2009, 3:01:06 PM5/10/09
to
Eric Gisse wrote on Thu, 07 May 2009 12:56:44 +0200:

> On May 7, 12:40 am, "Juan R."

> <juanREM...@canonicalscience.com> wrote:
>> Eric Gisse wrote on Wed, 06 May 2009 21:23:34 +0000:

(...)

>> Give me a citation of an author equating the scientific and rigorous


>> term "black hole candidate" with your ridiculous term "unicorn
>> candidate" or I will conclude that again you do not understand the
>> difference between rigorous scientific research and sci-fi movies or
>> between scientific knowledge and your own misconceptions.
>
> You missed the point.
>
> The object known as the "black hole" has specific predictions, all of
> which are satisfied by current observation in the instance of Sgr A*.

You missed the point that the same observations are compatible with
alternative models. Literature is extense, some references are

Evidence for Intrinsic Magnetic Moments in Black Hole Candidates 2002: Ap=
J
565, 447. Robertson S.L.; Leiter D.J.

Data for Sgr A* is reanalized here

http://arxiv.org/abs/astro-ph/0603746

> But since you seem to think there is still a fair amount of uncertainty
> (that you can't quantify), we might as well call it the unicorn
> candidate because it doesn't matter what is found you just won't be
> satisfied.

I see you failed to give a single citation to my above question.

(...)

--

juanR...@canonicalscience.com

unread,
May 10, 2009, 3:01:08 PM5/10/09
to
Eric Gisse wrote on Thu, 07 May 2009 12:56:44 +0200:

> On May 7, 12:40 am, "Juan R."

> <juanREM...@canonicalscience.com> wrote:
>> Eric Gisse wrote on Wed, 06 May 2009 21:23:34 +0000:

(...)

> How many documented examples of galaxies with supermassive dark objects


> pushing past a million solar masses would you like?

Precisely, the point is that! There exists many documented examples of
galaxies with supermassive dark objects, but zero documented examples tha=
t
one of them *is* a black hole. This is why the term "candidate to black
hole" continues being used in rigorous literature as in the next

S. Doeleman, et. al "Event horizon-scale structure in the supermassive
black hole candidate at the galactic center", Nature 455, 4 Sept. 2008, p
78.

>> > Ghez, et. al : "Stellar orbits around the galactic center black


>> > hole", ApJ 2-05 ; 744-757
>
> Actually it is ApJ 620. Usually I'm pretty good with proper
> transcription. That's why I write in pencil!
>
>
>> I would prefer a more rigorous "around the pressumed galactic center
>> black hole".
>
> Well I guess reading the title and immediately dismissing it _is_ an
> alternative to reading the abstract or even, god forbid, the entire
> article itself.

It is ironic that you make this accusation when in this same message you
dismissed a PRD paper just after claiming that cannot access it!

> If you have a problem with Ghez's methodology, I'd be glad to hear it.
> Otherwise, don't dismiss an article just because it doesn't play to your
> prejudices.

In absolutely no part that paper has showed the existence of a massive
black hole. Authors only only analized the confrontation to observations
of (i) a central massive black hole, (ii) a collection of black holes and
(iii) a cluster of neutron starts. Their conclusion was if it is not (ii)
or (iii) then may be (i). Just that!

Authors did not show the existence of specific properties would identify
the existence of a BH (e.g. horizon).

Authors do not considered other alternatives explaining the same
observations. Interesting analysis is in

Evidence for Intrinsic Magnetic Moments in Black Hole Candidates 2002: ApJ


565, 447. Robertson S.L.; Leiter D.J.

http://arxiv.org/abs/astro-ph/0603746

(...)

--

juanR...@canonicalscience.com

unread,
May 10, 2009, 3:01:10 PM5/10/09
to
Eric Gisse wrote on Fri, 08 May 2009 16:16:29 +0000:

> On May 7, 6:33 pm, "Juan R. Gonz�lez-�lvarez" <email.addr..
> .@not.given> wrote:
> [...]
>
>> The important part is that .rtf proves that your argument against
>> scalar radiation
>>
>> http://groups.google.com/group/sci.physics.foundations/msg/
>> bf1dcdb7d65dc3d1
>>
>> "Given that the pulsar orbits are decaying within 0.13 +/- 0.21%

>> according to Taylor, it sort-of puts the claim into a little bit
>> of doubt."
>>
>> was completely without basis.
>
> Uhhh, it is completely _with_ basis.
>
>>From the paper pulled off the conference site: "This is
> interestingly close to the expected value 0.735 % for the additional
> energy loss predicted for scalar gravitational radiation"
>
> Being off by a factor of 5 is interesting, just not helpful. Perhaps you
> have a more relevant reference?

You have misquoted, done the flase claims "factor 5" and deleted the
link in your message. I reintroduce it below [#] and give the complete
quotation from it.

The data by Weisberg & Taylor (2002) show that the excess of the orbital
period decrease relative to the predicted quadrupole energy loss is
\Delta_{obs} = (observed) - (quadrupole) = 0.78%. This is interestingly
close to the expected value 0.735% for the additional energy loss


predicted for scalar gravitational radiation

I.e. 0.78% is the difference between GR and observed value. Adding a
scalar contribution (not available in GR )gives

\Delta_{obs} = (observed) - (quadrupole) - (scalar) = 0.04%.

Once again your claims are without basis.

[#] http://ppc08.astro.spbu.ru/materials/thesis/50.rtf

--

Juan R.

unread,
May 10, 2009, 3:25:05 PM5/10/09
to
Eric Gisse wrote on Thu, 07 May 2009 12:56:44 +0200:

> On May 7, 12:40 am, "Juan R." González-Álvarez


> <juanREM...@canonicalscience.com> wrote:
>> Eric Gisse wrote on Wed, 06 May 2009 21:23:34 +0000:

(...)

>> http://arxiv.org/abs/0902.0346


>>
>>   "In summary, what our calculation suggests is that it might be  
>> possible to have a black hole without having a black hole."
>
> Only if you buy the assumption that vacuum polarization is relevant
> during collapse.

It is just the contrary, usual literature in BH does assumptions about
vacuum polarization. The authors *check* the validity of the assumption
used in BH literature.

Authors start from the Fulling-Sweeny-Wald theorem constraining
polarization to finite values and then *compute* the polarization,
obtaining appreciable deviation from a GR description of the collapse. The
GR description is based in the assumption that polarization is negligible.

> That's one of many major assumptions that are pretty flimsy.

As said it is not an assumption. Their assumptions are

(i) the quantum state is of the Hadamard form

(ii) the equivalence principle holds.

Which do you say is "pretty flimsy"?

(...)

--

CarlBrannen

unread,
May 13, 2009, 5:43:05 AM5/13/09
to
On May 7, 4:52 am, Gerry Quinn <ger...@indigo.ie> wrote:
> An issue still remains for me, though. Suppose we imagine that in our
> cube of simulated space, there is a grid of observers spread throughout
> it. Let us say they are made of whatever stuff exists at a higher
> energy level than the graviton cutoff, and they communicate with each
> other using high energy particles that travel at c and are completely
> unaffected by gravity. They watch whatever is going on as the black
> hole forms and thereafter, including experiments with infalling
> astronauts and whatnot, but they don't get involved.

I love how you've put these questions. This is the heart of the
matter.

If there is a flat space version of GR, then there is a flat space
metric. My guess is that it is Gullstrand-Painleve. Wikipedia
has a nice article:
http://en.wikipedia.org/wiki/Gullstrand-Painlev%C3%A9_coordinates

In Schwarzschild coordinates, stuff tossed into the black hole
approaches motionlessness as it approaches the event horizon.
This includes light, so it is natural to conclude that light is
slowed down in the presence of a black hole.

However, in GP coordinates it is more complicated. The speed
of light depends on its direction. While light exiting the black
hole is slowed down, light falling into the black hole is sped
up. The difference is because GP coordinates have a non
zero dr dt term. For light moving radially, putting
ds^2 d omega^2 = 0, and putting r=2m, one has:
ds^2 = -2dt dr - dr^2 = 0,
so
2 dr/dt = - (dr/dt)^2
which has two solutions, dr/dt = 0 for light which is, in the
limit, moving away from the black hole, and dr/dt = -2,
which corresponds to light moving into the black hole.

So in the higher energy regime, if gravitons are to support
GP coordinates all the way down to the event horizon, their
speed has to be at least 2c.

> What do the paths of infalling light rays in the interior region look
> like to them in their diagram? Are they moving forward in time, and
> what speed do they go?

More generally, at other radii, ds^2 = 0 for radially moving
(massless) particles satisfy
(1-2M/r) - 2 sqrt(2M/r) dr/dt - (dr/dt)^2 = 0.
which has two solutions:

-sqrt(2M/r) +1 and -sqrt(2M/r) -1

Inside the event horizon, both branches correspond to light
moving towards the singularity, but with two different speeds.
There speeds get larger and larger the closer you get to
the singularity. Assuming finite speed gravitons, at some
distance from the singularity the speed of light (and the limit
speed of matter) becomes faster than the speed of gravitons
and so physics has to break down. Where it breaks down
depends on the speed of gravitons (i.e. in the free space
limit far from the black hole).

Of course everything moves forward in time. And I've used
"c" to represent the speed of light far from the black hole,
as this is traditional.

Also, the above is under the assumption that spacetime
consists of only one Schwarzschild solution. It ignores the
presence of other matter in the universe. In the cosmological
case, the gravitons emitted by all the other matter in the universe
could effect the speed of light even in flat space but since
it's isotropic, the result far from nearby matter is still
Minkowski space.

> Another typical question: our grid of observers watch a black-hole
> jumping astronaut who starts falling from a point distant from the black
> hole at a certain time.

GP coordinates are particularly natural for keeping track of
the clocks on stuff falling into a black hole. The best description
of them is the article by Andrew J. S. Hamilton, Jason P. Lisle,
The River Model of Black Holes,
http://arxiv.org/abs/gr-qc/0411060
which extends to the Kerr metric.

Gerry Quinn

unread,
May 16, 2009, 6:12:59 AM5/16/09
to
In article <e0201372-3d33-44cf-81ff-
475fa4...@d19g2000prh.googlegroups.com>, ca...@brannenworks.com
says...

> On May 7, 4:52 am, Gerry Quinn <ger...@indigo.ie> wrote:
> > An issue still remains for me, though. Suppose we imagine that in our
> > cube of simulated space, there is a grid of observers spread throughout
> > it. Let us say they are made of whatever stuff exists at a higher
> > energy level than the graviton cutoff, and they communicate with each
> > other using high energy particles that travel at c and are completely
> > unaffected by gravity. They watch whatever is going on as the black
> > hole forms and thereafter, including experiments with infalling
> > astronauts and whatnot, but they don't get involved.
>
> I love how you've put these questions. This is the heart of the
> matter.

It certainly seems an obvious question to ask... it is disappointing
that nobody seems interested in answering it! Of course the hypothesis
of high-energy observers is a little fanciful in this context, even if
it is an exact description of what happens in laboratory situations
where we use effective field theories to describe the behaviour of (say)
phonons in a solid, and we can look at the solid all the while using
light if we wish.

But if our theory hypothesises an unexplained flat background, it may
obviously be assumed that there is really some physics behind this even
if we don't know what, and thus communication of the sort described
should presumably be available to whatever processes maintain it.

So, it's clearly reasonable to juxtapose the low energy solution and the
high energy background; and this should be easy enough if we use a
coordinate system suited to the latter.

> If there is a flat space version of GR, then there is a flat space
> metric. My guess is that it is Gullstrand-Painleve. Wikipedia
> has a nice article:
> http://en.wikipedia.org/wiki/Gullstrand-Painlev%C3%A9_coordinates

Since it's flat, why not just stick with Minkowski spacetime?


> In Schwarzschild coordinates, stuff tossed into the black hole
> approaches motionlessness as it approaches the event horizon.
> This includes light, so it is natural to conclude that light is
> slowed down in the presence of a black hole.

But am I correct in thinking that the Schwarzschild coordinates are in
fact those best suited to the grid of gravity-independent observers
operating in flat spacetime? Indeed, an observer distant from the black
hole is just one of these observers. The question then arises; how long
must one of these high energy observers, initially within the black
hole, wait for stuff tossed into the black hole to arrive at his
position?

If the answer to that question is "forever", or if the stuff has to go
to infinity in his time and come back reversed in time,then surely it is
untenable to assert that a solution based on a flat background but
replicating the GR interior solution is really compatible with the
assumptions underlying the concept of an effective field theory? Yet
that is what people have been saying here.

[--]

> > Another typical question: our grid of observers watch a black-hole
> > jumping astronaut who starts falling from a point distant from the black
> > hole at a certain time.
>
> GP coordinates are particularly natural for keeping track of
> the clocks on stuff falling into a black hole. The best description
> of them is the article by Andrew J. S. Hamilton, Jason P. Lisle,
> The River Model of Black Holes,
> http://arxiv.org/abs/gr-qc/0411060
> which extends to the Kerr metric.

But the question I'm asking is not about the astronaut's clock, but
about the movement of the astronaut - and indeed the changes in his
wristwatch - as seen/calculated by the gravity-independent observers
living in the flat background spacetime. The astronaut's clock can be
monitored by them, of course; they need merely observe his wristwatch,
and make the appropriate corrections.

- Gerry Quinn

Juan R.

unread,
May 17, 2009, 2:48:23 PM5/17/09
to
CarlBrannen wrote on Wed, 13 May 2009 09:43:05 +0000:

> If there is a flat space version of GR, then there is a flat space
> metric.

GR is a curved spacetime theory. Of course, it is possible to reformulate
it spliting spacetime metric as g_ab = \eta_ab + h_ab, but that \eta_ab
has not physical meaning in GR. I agree with Straumann that flat spacetime
is "a kind of unobservable aether" in GR.

(...)

> Inside the event horizon, both branches correspond to light moving
> towards the singularity, but with two different speeds. There speeds get
> larger and larger the closer you get to the singularity. Assuming finite
> speed gravitons, at some distance from the singularity the speed of
> light (and the limit speed of matter) becomes faster than the speed of
> gravitons and so physics has to break down. Where it breaks down depends
> on the speed of gravitons (i.e. in the free space limit far from the
> black hole).

If you are working within GR, then there are not gravitons (in the sense
of particle physics). First, GR cannot be consistently quantized. Second,
there is not positive EMT associated to the gravitons, those gravitons
cannot be associated to gravitational forces (there is not gravitational
forces in GR)...

If you are working within a quantum field theory over flat spacetime, then
there are real gravitons (in the sense of particle physics). Those
gravitons has positive EMT, and are associated to true gravitational
forces over a flat background so observable as the background of the
Standard Model... However, in flat spacetime theory there is not
singularities because the EMT of the graviton field (EMT absent in GR)
prevents the formation of singularities and event horizons.

Regards.

CarlBrannen

unread,
May 18, 2009, 2:43:19 AM5/18/09
to
On May 16, 3:12�am, Gerry Quinn <ger...@indigo.ie> wrote:
> > If there is a flat space version of GR, then there is a flat space
> > metric. My guess is that it is Gullstrand-Painleve. Wikipedia
> > has a nice article:
> >http://en.wikipedia.org/wiki/Gullstrand-Painlev%C3%A9_coordinates
>
> Since it's flat, why not just stick with Minkowski spacetime?

You can use Minkowski spacetime for the high energy observers,
but it may need a different speed for its "light", or whatever
particle
is used for communication.

By the way, this sort of thing, where a space has two different
speeds, each of which is associated with a Minkowski coordinate
system, is similar to what happens when you look at the small
amplitude waves / vibrations in an isotropic 3-dimensional solid.

There are two types of waves in that circumstance, transverse
and longitudinal and they travel with two different wave speeds.
For earthquakes, they are known as S and P waves where
S (or transverse) stands for secondary because they arrive
after the primary waves.

But with each of these waves, at least for an isotropic solid,
the wave equation is the massless Klein-Gordon that is
used for massless relativistic spinless particles. They just
have two different wave speeds.

> > In Schwarzschild coordinates, stuff tossed into the black hole
> > approaches motionlessness as it approaches the event horizon.
> > This includes light, so it is natural to conclude that light is
> > slowed down in the presence of a black hole.
>
> But am I correct in thinking that the Schwarzschild coordinates are in
> fact those best suited to the grid of gravity-independent observers
> operating in flat spacetime? �Indeed, an observer distant from the black
> hole is just one of these observers.

To get Gullstrand-Painleve (GP) coordinates from Schwarzschild
one converts events (x,y,z,t) to (x,y,z,t'). That is, the only change
is the assignment of coordinate times for events. In that sense,
they are equally suited models of a black hole.

Schwarzschild coordinates differ from GP coordinates mostly in
that Schwarzschild have no dr dt term in the metric. This means
that the speed of light in Schwarzschild coordinates does not
depend on direction away from or towards the black hole. This
follows from the fact that you can replace dr with -dr and
the metric is unchanged.

> The question then arises; how long must one of these
> high energy observers, initially within the black hole,
> wait for stuff tossed into the black hole to arrive at his
> position?

In GP coordinates, the coordinate time to reach the
singularity from just outside the event horizon is finite.

> If the answer to that question is "forever", or if the stuff
> has to go to infinity in his time and come back reversed
> in time,then surely it is untenable to assert that a solution
> based on a flat background but replicating the GR interior
> solution is really compatible with the assumptions
> underlying the concept of an effective field theory? �Yet
> that is what people have been saying here.

I think I agree with you here. If we postulate that gravity
is a force operating on a flat background (along with all
the other forces), then it should be possible to describe
stuff that happens in the universe in a manner compatible
with the kind of stuff that happens in a flat space.
That means that wormholes, weird reversals of time and
space, and that sort of thing are just mathematics, they
are not reality.

> > GP coordinates are particularly natural for keeping track of
> > the clocks on stuff falling into a black hole. The best description
> > of them is the article by Andrew J. S. Hamilton, Jason P. Lisle,
> > The River Model of Black Holes,
> >http://arxiv.org/abs/gr-qc/0411060
> > which extends to the Kerr metric.
>
> But the question I'm asking is not about the astronaut's clock, but
> about the movement of the astronaut - and indeed the changes in his
> wristwatch - as seen/calculated by the gravity-independent observers
> living in the flat background spacetime. �The astronaut's clock can be
> monitored by them, of course; they need merely observe his wristwatch,
> and make the appropriate corrections.

My interpretation of the time coordinate of GP coordinates is that
they give the outside observer's value for the event. That this number
has something to do with what shows up on the astronaut's watch
is only coincidental.

The coincidence is interpreted in the "River Model of Black Holes"
as a river of spacetime moving towards the singularity. Then the
falling astronaut isn't sped up and so his wristwatch is not
effected by time dilation. This is an interesting interpretation
of the physics of gravity, I think, but I can't tell you what the
coincidence means.

My reason for picking GP coordinates is that they seem to
be preferred in the context of geometric algebra, as I mentioned
earlier i nthis thread.

Gerry Quinn

unread,
May 18, 2009, 12:19:56 PM5/18/09
to
In article <f53b2596-b030-4fdd-a6e7-28052cfdccd2
@b6g2000pre.googlegroups.com>, ca...@brannenworks.com says...

> On May 16, 3:12�am, Gerry Quinn <ger...@indigo.ie> wrote:
> > > If there is a flat space version of GR, then there is a flat space
> > > metric. My guess is that it is Gullstrand-Painleve. Wikipedia
> > > has a nice article:
> > >http://en.wikipedia.org/wiki/Gullstrand-Painlev%C3%A9_coordinates
> >
> > Since it's flat, why not just stick with Minkowski spacetime?
>
> You can use Minkowski spacetime for the high energy observers,
> but it may need a different speed for its "light", or whatever
> particle
> is used for communication.

That's no problem, so long as it is constant (i.e. unaffected by
gravity).

> By the way, this sort of thing, where a space has two different
> speeds, each of which is associated with a Minkowski coordinate
> system, is similar to what happens when you look at the small
> amplitude waves / vibrations in an isotropic 3-dimensional solid.

Indeed, but in this case we are not interested in the detailed
characteristics of the 'high-energy' waves - we just assume their speed
to be constant and large. (As if we were watching waves on the ocean
through the medium of light, for example.)

> > > In Schwarzschild coordinates, stuff tossed into the black hole
> > > approaches motionlessness as it approaches the event horizon.
> > > This includes light, so it is natural to conclude that light is
> > > slowed down in the presence of a black hole.
> >
> > But am I correct in thinking that the Schwarzschild coordinates are in
> > fact those best suited to the grid of gravity-independent observers
> > operating in flat spacetime? �Indeed, an observer distant from the black
> > hole is just one of these observers.
>
> To get Gullstrand-Painleve (GP) coordinates from Schwarzschild
> one converts events (x,y,z,t) to (x,y,z,t'). That is, the only change
> is the assignment of coordinate times for events. In that sense,
> they are equally suited models of a black hole.
>
> Schwarzschild coordinates differ from GP coordinates mostly in
> that Schwarzschild have no dr dt term in the metric. This means
> that the speed of light in Schwarzschild coordinates does not
> depend on direction away from or towards the black hole. This
> follows from the fact that you can replace dr with -dr and
> the metric is unchanged.

But these 'high-energy observers' have a specific coordinate system
which will simply be the time in some agreed upon inertial frame.
Without loss of generality, it can even be the same as Earth time (if we
assume they consider that special and general relativistic influences on
Earth time are insignificant). Since spacetime for them is flat, they
have no difficulty in relating clock times to those in a given arbitrary
inertial frame, just as we do. Unlike us, they can do it just as easily
from the centre of a black hole.

They don't have any reason to choose a different coordinate system, any
more than we do in any situation in which we have no reason to believe
our measuring rods and clocks are distorted by effects other than the
straightforward ones associated with special relativity. Remember, they
are observing, not theorising. They see stuff falling into black holes,
and simply monitor what happens to it, e.g. they see it either never
getting to the horizon, or falling right through it, or whatever.

So their natural description of black hole geometry as seen by ordinary
matter and light, insofar as GR remains valid, should be the
Schwarzschild solution, no? Of course if this is the case, the obvious
interpretation is that GR is valid only outside the horizon.

So why don't we just do some funky coordinate changes and brush aside
the problems of the Schwarzschild solution? This, after all, is
repeatedly touted as the greatest advance of the twentieth century in
understanding black holes! The reason we have problems doing it here is
simple - we have hypothesised a flat background spacetime, and the
Schwarzschild coordinates are the natural ones in which to describe the
physical system. Here the singularity at the horizon has a very natural
physical interpretation too. In this interpretation, stuff that falls in
doesn't ever hit the horizon; it stays there and since this is the point
at which GR breaks down, it eventually discovers some new physics.
(After a very long time in terms of the high-energy observers' clocks,
but very quickly in terms of the clocks of somebody falling into a black
hole.)

Of course we *can* still do our coordinate changes and hypothesise that
GR still works through and inside the horizon. But now we have the
problems of reconciling the GR interior solution with that of observers
unaffected by gravity. The best you can say is that it gets very weird;
you are trying to reconcile a picture in which space and time and
switched with one in which space and time are just the same as on Earth.
And you still have done nothing to fix the central singularity. Let GR
break down at the Schwarzschild radius, and the central singularity is
no longer an issue. (Which is good, because the central singularity
seems just about unfixable. Shouldn't that be telling us we made a
mistake somewhere before we got there?)

So much simpler to assume that GR breaks down at the horizon, isn't it?
The high energy observers have a simple explanation to offer, too - they
point out that infalling stuff, according to their observations, is
stuck indefinitely near the horizon. Very rare high-energy interactions
- again unaffected by gravity - will become frequent in terms of the
clocks on the infalling stuff, which have (according to the high-energy
observers) effectively stopped. Hence, the infalling observers will see
new physics.

> > The question then arises; how long must one of these
> > high energy observers, initially within the black hole,
> > wait for stuff tossed into the black hole to arrive at his
> > position?
>
> In GP coordinates, the coordinate time to reach the
> singularity from just outside the event horizon is finite.
>
> > If the answer to that question is "forever", or if the stuff
> > has to go to infinity in his time and come back reversed
> > in time,then surely it is untenable to assert that a solution
> > based on a flat background but replicating the GR interior
> > solution is really compatible with the assumptions
> > underlying the concept of an effective field theory? �Yet
> > that is what people have been saying here.
>
> I think I agree with you here. If we postulate that gravity
> is a force operating on a flat background (along with all
> the other forces), then it should be possible to describe
> stuff that happens in the universe in a manner compatible
> with the kind of stuff that happens in a flat space.
> That means that wormholes, weird reversals of time and
> space, and that sort of thing are just mathematics, they
> are not reality.

That's the key point. As I see it, you can have a flat background, or
you can have GR black holes. But it seems rather difficult to believe
in both when you take the implications seriously.

- Gerry Quinn

Eric Gisse

unread,
May 18, 2009, 5:37:16 PM5/18/09
to
On May 10, 11:25 am, "Juan R." Gonz�lez-�lvarez

<juanREM...@canonicalscience.com> wrote:
> Eric Gisse wrote on Thu, 07 May 2009 12:56:44 +0200:
>
> > On May 7, 12:40 am, "Juan R." Gonz�lez-�lvarez

> > <juanREM...@canonicalscience.com> wrote:
> >> Eric Gisse wrote on Wed, 06 May 2009 21:23:34 +0000:
>
> (...)
>
> >>http://arxiv.org/abs/0902.0346
>
> >> "In summary, what our calculation suggests is that it might be
> >> possible to have a black hole without having a black hole."
>
> > Only if you buy the assumption that vacuum polarization is relevant
> > during collapse.
>
> It is just the contrary, usual literature in BH does assumptions about
> vacuum polarization. The authors *check* the validity of the assumption
> used in BH literature.>
> Authors start from the Fulling-Sweeny-Wald theorem constraining
> polarization to finite values and then *compute* the polarization,
> obtaining appreciable deviation from a GR description of the collapse. The
> GR description is based in the assumption that polarization is negligible.

More like "non-existent". Classical general relativity doesn't concern
itself with quantum mechanics.

>
> > That's one of many major assumptions that are pretty flimsy.
>
> As said it is not an assumption. Their assumptions are
>
> (i) the quantum state is of the Hadamard form
>
> (ii) the equivalence principle holds.
>
> Which do you say is "pretty flimsy"?

The assumption you purposefully left out which forms the fundamental
thesis of the paper - that the vaccum polarization energy is large
during collapse. I'm also skeptical at the thought of slaping a bra
and ket onto the field equations and calling it a day, given the
nonlinearities of the theory. But I'll let that one be.

The argument can be substituted for other forces that manage to prop
the star up an epsilon away from forming a trapped surface, eg: quark
stars. Except there isn't any _observation_ supporting this concept.

Plus I'm rather amused that a state that is _explicitly known_ to be
singular at the horizon manages to give a result that diverges as one
approaches the horizon. A stress tensor that diverges as one
approaches the event horizon gives large contributions to the stress
tensor as one approaches the horizon?! Imagine that.

Perhaps you could explain why you think this is observationally
relevant.

>
> (...)
>
> --http://www.canonicalscience.org/
>
> Usenet Guidelines:http://www.canonicalscience.org/en/miscellaneouszone/guidelines.html

Eric Gisse

unread,
May 18, 2009, 5:57:10 PM5/18/09
to
On May 10, 11:01�am, juanREM...@canonicalscience.com wrote:
> Eric Gisse wrote on Thu, 07 May 2009 12:56:44 +0200:
>
> > On May 7, 12:40 am, "Juan R."
> > <juanREM...@canonicalscience.com> wrote:
> >> Eric Gisse wrote on Wed, 06 May 2009 21:23:34 +0000:
>
> (...)
>
> >> Give me a citation of an author equating the scientific and rigorous
> >> term "black hole candidate" with your ridiculous term "unicorn
> >> candidate" or I will conclude that again you do not understand the
> >> difference between rigorous scientific research and sci-fi movies or
> >> between scientific knowledge and your own misconceptions.
>
> > You missed the point.
>
> > The object known as the "black hole" has specific predictions, all of
> > which are satisfied by current observation in the instance of Sgr A*.
>
> You missed the point that the same observations are compatible with
> alternative models. Literature is extense, some references are
>
> Evidence for Intrinsic Magnetic Moments in Black Hole Candidates 2002: Ap=
> J
> 565, 447. Robertson S.L.; Leiter D.J.
>
> Data for Sgr A* is reanalized here
>
> http://arxiv.org/abs/astro-ph/0603746

"The assumptions on which these calculations for Sgr A* were based are
as follows: (1)
A hard surface, possibly highly redshifted, ( 2Rg < R < 100Rg) assumed
to exist within Sgr
A* [...]"

Didn't we JUST COVER THIS? It doesn't even _matter_ if the analysis
supporting the object called "MECO" is good (it isn't) - the primary
assumption that there's some "surface" is completely unsupported by
observation. VLBI imagery put the upper bound on the surface beyond
the critical radius implied by Buchdhal's theorem.

This surface they assume to exist...doesn't.

>
> > But since you seem to think there is still a fair amount of uncertainty
> > (that you can't quantify), we might as well call it the unicorn
> > candidate because it doesn't matter what is found you just won't be
> > satisfied.
>
> I see you failed to give a single citation to my above question.

Probably because the "questions" are contemptable.

The "alternative" remains as such if 15 years of observation of Sgr A*
are discarded. The gravastar concept is just like the MECO concept,
but more ridiculous. I asked if you had actually read about the object
because I find it odd that you rail against black holes at every
opportunity but seem to be "for" an object that's a layer cake of
unstable and impossible to form layers of matter and weirdly-depleted
regions of spacetime.

>
> (...)
>
> --http://www.canonicalscience.org/
>
> Usenet Guidelines:http://www.canonicalscience.org/en/miscellaneouszone/guidelines.html

Juan R.

unread,
May 19, 2009, 12:00:08 PM5/19/09
to
Eric Gisse wrote on Mon, 18 May 2009 21:57:10 +0000:

> On May 10, 11:0 am,


juanREM...@canonicalscience.com
> wrote:
>> Eric Gisse wrote on Thu, 07 May 2009 12:56:44 +0200:
>>
>> > On May 7, 12:40 am, "Juan R."
>> > <juanREM...@canonicalscience.com> wrote:
>> >> Eric Gisse wrote on Wed, 06 May 2009 21:23:34 +0000:
>>
>> (...)
>>
>> >> Give me a citation of an author equating the scientific and rigorous
>> >> term "black hole candidate" with your ridiculous term "unicorn
>> >> candidate" or I will conclude that again you do not understand the
>> >> difference between rigorous scientific research and sci-fi movies or
>> >> between scientific knowledge and your own misconceptions.

(...)

>> You missed the point that the same observations are compatible with
>> alternative models. Literature is extense, some references are
>>
>> Evidence for Intrinsic Magnetic Moments in Black Hole Candidates 2002:

>> ApJ 565, 447. Robertson S.L.; Leiter D.J.


>>
>> Data for Sgr A* is reanalized here
>>
>> http://arxiv.org/abs/astro-ph/0603746
>
> "The assumptions on which these calculations for Sgr A* were based are
> as follows: (1)
> A hard surface, possibly highly redshifted, ( 2Rg < R < 100Rg) assumed
> to exist within Sgr
> A* [...]"
>
> Didn't we JUST COVER THIS? It doesn't even _matter_ if the analysis
> supporting the object called "MECO" is good (it isn't)

Again you do a seletive quotation. A more complete quotation from the pdf
is:

"Perhaps the strongest claimed evidence for an event horizon in any
black hole candidate is the one made for Sgr A* (Broderick and Narayan
2006, hereafter BN06) based on its low radiated flux in the near
infrared.

BN06 considered surface thermal radiations from an object with radius
in the range 2Rg < R < 100Rg, where Rg = GM/c2 ~= 5.5 x 10^11 cm for a
mass of ~ 3.7 x 10^6 M_{circleddot}. Subject to three critical
assumptions they showed that if redshifted, hard surface thermal
emissions at 3.8 microm were produced from such an accretion rate the
radiated flux would be too high unless the source radius were larger
than about 40Rg . But since the 3.5mm emissions of the compact radio
source apparently originate within ~ 10-20Rg of the central object
(Shen et al. 2005, Bower et al. 2004), one would expect the NIR to
originate within the same region. It is possible that future VLBI
measurements will further constrain the size of the emitting region.
The smaller the region, the more severe the constraint on thermal
emissions from the mass accretion rate. If confined to the
Schwarzschild diameter, the accretion rate in accord with the
assumptions of BN06 would have to be less than about 3 x 10^13 g/s (5 x
10^-13 M_{circledot} /yr).

The assumptions on which these calculations for Sgr A* were based are
as follows: (1) A hard surface, possibly highly redshifted, ( 2Rg < R <
100Rg) assumed to exist within Sgr A* [...]"

> - the primary


> assumption that there's some "surface" is completely unsupported by
> observation. VLBI imagery put the upper bound on the surface beyond the
> critical radius implied by Buchdhal's theorem.
>
> This surface they assume to exist...doesn't.

Previous claims of existence of an event horizon was based in the study of
exactly the same region (2Rg < R < 100Rg) as showed in above quotation (in
the part you sniped).

Author repeated the analysis of the *same* region and found that data is
consistent with a model without the assumption of a black hole.

Moreover, the authors also remark (in the part you sniped) how VLBI
measurements would give accretion rates less than 10^13 for the horizon
radius in a BH model. The accepted value for SgrA* is (Baganoff et al.
2003): ~ 10^20.

Moreover magnetic moments associated to the surface have been also
recently detected in other black holes candidates. They give several
references, you also omit.

>> > But since you seem to think there is still a fair amount of
>> > uncertainty (that you can't quantify), we might as well call it the
>> > unicorn candidate because it doesn't matter what is found you just
>> > won't be satisfied.
>>
>> I see you failed to give a single citation to my above question.
>
> Probably because the "questions" are contemptable.

You failed because you could find zero citations supporting your
ridiculous claim about the existence of an unicorn in Sgr A* :-D

> The "alternative" remains as such if 15 years of observation of Sgr A*
> are discarded. The gravastar concept is just like the MECO concept, but
> more ridiculous. I asked if you had actually read about the object

Previously, I asked you to sustain a series of your claims and
acussations. You just avoided to reply.

Not just you do not retract but add some new now. I will do a new public
plea to you to support yours:

(i)
I gave you a reference of one renowned expert in BH with papers in
gravastars. Do you maintain now he is working in a "ridiculous" concept
because has not read about it? Or do you maintain now he is working in an
"alternative" that remains only because he is discharding 15 years of
observation?

(ii)
I have also given you a list of papers in gravastars published in Class.
Quan. Grav. journal, including a recent paper from 2009.

Do you maintain now that referees are accepting papers in "ridiculous"
concepts because have not read about it? Or do you maintain now referees
are accepting papers in "alternative" because are discharding 15 years of
observation?

(iii)
I invited you to do a detailed analysis of the model given in the
reference I introduced earlier in this thread. After your strong claims
everyone following this thread waited a discussion of his pros and cons.
But once again you avoided to do it.

> because I find it odd that you rail against black holes at every
> opportunity

In previous messages you misquoted references. Above you have misquoted
another reference. Now you misquote me.

Several times before in this thread I have *suggested the use* of the term
"black hole candidate" to refer to those objects needing further study to
check if they are or are not black holes. I have said this to you three or
four times before, thus either you do not read messages or you
deliberately ignore I am saying.


--

Eric Gisse

unread,
May 20, 2009, 12:06:12 PM5/20/09
to
On May 19, 8:00�am, "Juan R." Gonz�lez-�lvarez

No, the strongest evidence is direct VLBI imagery of Sgr A*, and the
next strongest evidence is kinematic data of the 20 or so stars within
a parsec of the black hole associated with Sgr A*.

>
> � �BN06 considered surface thermal radiations from an object with radius


> � �in the range 2Rg < R < 100Rg, where Rg = GM/c2 ~= 5.5 x 10^11 cm for a
> � �mass of ~ 3.7 x 10^6 M_{circleddot}. Subject to three critical
> � �assumptions they showed that if redshifted, hard surface thermal
> � �emissions at 3.8 microm were produced from such an accretion rate the
> � �radiated flux would be too high unless the source radius were larger
> � �than about 40Rg . But since the 3.5mm emissions of the compact radio
> � �source apparently originate within ~ 10-20Rg of the central object
> � �(Shen et al. 2005, Bower et al. 2004), one would expect the NIR to
> � �originate within the same region. It is possible that future VLBI
> � �measurements will further constrain the size of the emitting region.

That there's an offset between the black hole proper and the emitting
region isn't especially new, though Doeleman makes the argument more
credible.

I didn't paste this because _it is not relevant_.

> � �The smaller the region, the more severe the constraint on thermal


> � �emissions from the mass accretion rate. If confined to the
> � �Schwarzschild diameter, the accretion rate in accord with the
> � �assumptions of BN06 would have to be less than about 3 x 10^13 g/s (5 x
> � �10^-13 M_{circledot} /yr).

Our understanding of the accretion disk region is ridiculously weak,
the best we can guess is that it is torodial/annular in shape.

>
> � �The assumptions on which these calculations for Sgr A* were based are


> � �as follows: (1) A hard surface, possibly highly redshifted, ( 2Rg < R <
> � �100Rg) assumed to exist within Sgr A* [...]"
>
> > - the primary
> > assumption that there's some "surface" is completely unsupported by
> > observation. VLBI imagery put the upper bound on the surface beyond the
> > critical radius implied by Buchdhal's theorem.
>
> > This surface they assume to exist...doesn't.
>
> Previous claims of existence of an event horizon was based in the study of
> exactly the same region (2Rg < R < 100Rg) as showed in above quotation (in
> the part you sniped).

The paper PREDATES Doeleman's VLBI study. Furthermore, _there is no
hard surface_.

Flaring from infalling matter in the IR region has been observed, but
nothing associated with an impact on a surface.

What is complicated about this concept?

I think you are arguing simply because you don't like black holes, and
will advocate for anything that is marginally consistent with
observation regardless of how absurd it is because you can't accept
the possibility that black holes are real constructs.

If you want to argue that MECOs or gravastars are real, you are going
to have to do a HELL of a lot better than "well...it is consistent
with observation".

Why?

Because a MECO requires hydostatic equilibrium [a very unstable one]
of matter in a configuration that we have no reason to think exists
that is furthermore supported by Eddington luminosity, which is not
seen.

Did you even read the paper about the gravastar? Tell me, honestly, if
you think it is a more "natural", "acceptable", or "likely"
alternative to a black hole.

>
> Author repeated the analysis of the *same* region and found that data is
> consistent with a model without the assumption of a black hole.

How contrived does the explanation have to be before someone can slice
it apart with Occam's razor?

>
> Moreover, the authors also remark (in the part you sniped) how VLBI
> measurements would give accretion rates less than 10^13 for the horizon
> radius in a BH model. The accepted value for SgrA* is (Baganoff et al.
> 2003): ~ 10^20.

Wow, one model for accretion flow is wrong. BFD.

Saying the accretion rate is more than a particular author expects
isn't the most compelling of argument.

>
> Moreover magnetic moments associated to the surface have been also
> recently detected in other black holes candidates. They give several
> references, you also omit.

It sounds like the only way you will be satisfied is if I copy and
paste the entire article so that nothing is 'left out'. At any rate, I
expect that you aren't terribly interested in discussing the merits of
the acronym soup of a paper. Let's find out.

Start from the top: False goddamn dichotomy. The key is how your form
the question.

The authors form the paper in a way such that it appears that you can
have _either_ a magnetic moment _or_ a black hole, but not both. If
you honestly believe that there are not magnetic fields associated
with relativistic plasmas, I have some introductory plasma and space
physics textbooks for you to read.

I know these folks have their theory and they are looking at it from
that angle. That's obvious. But what you are missing is that they are
only considering things that make their idea make sense. They only
consider _two_ models of accretion flow: Bondi flow (spherical
accretion) and an "optically thick, geometrically thin accretion
disk".

I take great offense to "science by exclusion", especially when the
exclusion is to cherry picked models. Bondi flow is clearly wrong -
there ain't no spherical symmetry. As far as the other model is
concerned, judging from what I've read, it has been known for a fairly
long time that the model is wrong given the cited (in the MECO
reference, as well as others) many orders of magnitude wrongness in
luminosity.

I actually find it rather curious that the authors do not even
seriously consider the RAIF model, which by the way is the generally
accepted model, of accretion [ http://arxiv.org/abs/astro-ph/0304099 ]
until midway through the paper in order to dismiss it using
"observations" of magnetic fields that are based upon polarization of
radio waves. That particular argument was not put together all that
well, in my opinion.

The entire paper is based upon observed luminosities and radio wave
polarization modes, neither of which get anywhere close to showing
what they try to show. There's a reason the MECO concept struggles to
find traction...anywhere.

>
> >> > But since you seem to think there is still a fair amount of
> >> > uncertainty (that you can't quantify), we might as well call it the
> >> > unicorn candidate because it doesn't matter what is found you just
> >> > won't be satisfied.
>
> >> I see you failed to give a single citation to my above question.
>
> > Probably because the "questions" are contemptable.
>
> You failed because you could find zero citations supporting your
> ridiculous claim about the existence of an unicorn in Sgr A* :-D
>
> > The "alternative" remains as such if 15 years of observation of Sgr A*
> > are discarded. The gravastar concept is just like the MECO concept, but
> > more ridiculous. I asked if you had actually read about the object
>
> Previously, I asked you to sustain a series of your claims and
> acussations. You just avoided to reply.

I actually forgot about this thread until you decided to start calling
me a "crank" using this thread as a reference. Sore loser.

>
> Not just you do not retract but add some new now. I will do a new public
> plea to you to support yours:
>
> (i)
> I gave you a reference of one renowned expert in BH with papers in
> gravastars. Do you maintain now he is working in a "ridiculous" concept
> because has not read about it?

Why, yes I do still think it is ridiculous. Don't you?

Does the so-called "renowned expert", whom I have never heard of,
actually think his model is a serious contender or is it a toy model
he is pursuing for his own particular reasons?

>Or do you maintain now he is working in an
> "alternative" that remains only because he is discharding 15 years of
> observation?

This would be an example of a ridiculous question formulated to make
me look bad. It is nothing of the sort, and you know it.

>
> (ii)
> I have also given you a list of papers in gravastars published in Class.
> Quan. Grav. journal, including a recent paper from 2009.
>
> Do you maintain now that referees are accepting papers in "ridiculous"
> concepts because have not read about it? Or do you maintain now referees
> are accepting papers in "alternative" because are discharding 15 years of
> observation?

More ridiculous questions designed to make me look bad.

Class. Quan. Grav has published all sorts of interesting ideas over
the years. The journal is not one that is primarily concerned with
observation. That's for other journals like ApJ, Nature, etc. It is a
journal for theoretical exercises - like MECO's.

>
> (iii)
> I invited you to do a detailed analysis of the model given in the
> reference I introduced earlier in this thread. After your strong claims
> everyone following this thread waited a discussion of his pros and cons.
> But once again you avoided to do it.

Yes, the demand has been great. My inbox was filling with requests,
and the interest in the thread is extraordinary. Oh wait, no, I forgot
about this thread and was only reminded about it when you decided to
reference it in your latest series of whines about me in sci.physics /
sci.physics.relativity about how I'm a crank.

I'm still waiting for you to express a concrete opinion on the MECO/
gravastar concept. Do you actually think they are reasonable models
for the world despite their instabilities and severely contrived
nature?

http://arxiv.org/abs/0709.0532

Unstable for spin parameter >~ 0.4. Oops.

http://www.iop.org/EJ/abstract/0004-637X/652/1/518

"Based on a spectral analysis of the X-ray continuum that employs a
fully relativistic accretion disk model, we conclude that the compact
primary of the binary X-ray source GRS 1915+105 is a rapidly rotating
Kerr black hole. We find a lower limit on the dimensionless spin
parameter of a* > 0.98."

OOPS.

http://arxiv.org/abs/0803.4200

"Therefore, although the existence of gravastars cannot be excluded
from such dynamical models, our results do indicate that, even if
gravastars indeed exist, they do not exclude the existence of black
holes."

OOPS.

What hoops shall I jump through next?

>
> > because I find it odd that you rail against black holes at every
> > opportunity
>
> In previous messages you misquoted references. Above you have misquoted
> another reference. Now you misquote me.
>
> Several times before in this thread I have *suggested the use* of the term
> "black hole candidate" to refer to those objects needing further study to
> check if they are or are not black holes. I have said this to you three or
> four times before, thus either you do not read messages or you
> deliberately ignore I am saying.

Or I think what you are saying is wrong.

Crazy thought, I know! But that's how I roll.

>
> --http://www.canonicalscience.org/
>
> Usenet Guidelines:http://www.canonicalscience.org/en/miscellaneouszone/guidelines.html

juanR...@canonicalscience.com

unread,
May 21, 2009, 4:29:51 PM5/21/09
to
Eric Gisse wrote on Mon, 18 May 2009 21:37:16 +0000:

> On May 10, 11:25 am, "Juan R." Gonzalez-Alvarez


> <juanREM...@canonicalscience.com> wrote:
>> Eric Gisse wrote on Thu, 07 May 2009 12:56:44 +0200:
>>

>> > On May 7, 12:40 am, "Juan R." Gonzalez-Alvarez


>> > <juanREM...@canonicalscience.com> wrote:
>> >> Eric Gisse wrote on Wed, 06 May 2009 21:23:34 +0000:
>>
>> (...)
>>
>> >>http://arxiv.org/abs/0902.0346
>>
>> >> "In summary, what our calculation suggests is that it might be
>> >> possible to have a black hole without having a black hole."
>>
>> > Only if you buy the assumption that vacuum polarization is relevant
>> > during collapse.
>>
>> It is just the contrary, usual literature in BH does assumptions about

>> vacuum polarization. The authors *check* the validity of the assumptio=


n
>> used in BH literature.>
>> Authors start from the Fulling-Sweeny-Wald theorem constraining
>> polarization to finite values and then *compute* the polarization,
>> obtaining appreciable deviation from a GR description of the collapse.
>> The GR description is based in the assumption that polarization is
>> negligible.
>
> More like "non-existent". Classical general relativity doesn't concern
> itself with quantum mechanics.

I am sure that any reader of this newsgroup knows that general relativity
is a classical theory. Your straw man avoided my main point.

The general relativity description is based in the assumption that the
collapse can be completely understood in classical terms and that no
relevant quantum corrections arise. Evidently, using GR you cannot
evaluate the size of the quantum corrections and no way to test if the
assumption of classicality holds or not.

Authors of above paper *computed* quantum field corrections and found
deviations from a purely GR description of the collapse.

This is not very different from classical electrodynamics being corrected
by quantum electrodynamics. Evidently the correction terms are obtained
from the quantum theory.

But you think that no correction to GR may be waited from quantum theory,
right?

>> > That's one of many major assumptions that are pretty flimsy.
>>
>> As said it is not an assumption. Their assumptions are
>>
>> (i) the quantum state is of the Hadamard form
>>
>> (ii) the equivalence principle holds.
>>
>> Which do you say is "pretty flimsy"?
>
> The assumption you purposefully left out which forms the fundamental
> thesis of the paper - that the vaccum polarization energy is large
> during collapse.

This plain wrong statement was corrected before. They do not assume some
value for the polarization, they compute the polarization in the section
"4. Our specific calculation" and found is not zero.

It is the GR description which is based in the *assumption* that
polarization is zero.

Just as waited you have not confirmed which of their assumptions in the
paper is "pretty flimsy".

> I'm also skeptical at the thought of slaping a bra and ket onto the
> field equations and calling it a day, given the nonlinearities of the
> theory. But I'll let that one be.

Could someone parse this for me please?

> The argument can be substituted for other forces that manage to prop th=
e
> star up an epsilon away from forming a trapped surface, eg: quark stars=
.


> Except there isn't any _observation_ supporting this concept.
>
> Plus I'm rather amused that a state that is _explicitly known_ to be
> singular at the horizon manages to give a result that diverges as one
> approaches the horizon.

We may be still more amused because they give a non-divergent result as
one approaches the horizon :-D

> A stress tensor that diverges as one approaches the event horizon gives

> large contributions to the stress tensor as one approaches the horizon?=
!
> Imagine that.

I am trying to imagine how you can do this kind of false claims without
waiting a response!

For a Hadamard quantum state, the renormalized stress-energy tensor is
automatically *finite* by the Fulling-Sweeny-Wald theorem.

Moreover, everyone can access the above pdf and find after their equation
(4.35):

"However, the renormalised stress-energy-momentum tensor gives an
arbitrarily large (albeit finite) energy-condition-violating
contribution to the right hand side of the semiclassical Einstein
equations as the horizon formation condition 2M/r=1 is approached."

> Perhaps you could explain why you think this is observationally
> relevant.

It is evident. I give the response in a previous message. It is still
quoted at the start of this message.

--

Eric Gisse

unread,
May 22, 2009, 9:35:33 AM5/22/09
to
On May 21, 12:29�pm, juanREM...@canonicalscience.com wrote:

[...]

> Moreover, everyone can access the above pdf and find after their equation
> (4.35):
>
> � "However, the renormalised stress-energy-momentum tensor gives an
> � �arbitrarily large (albeit finite) energy-condition-violating
> � �contribution to the right hand side of the semiclassical Einstein
> � �equations as the horizon formation condition 2M/r=1 is approached."

Key words _arbitrarily large_. In other words, it can be _whatever you
want it to be_. This plus the increased arbitrariness (and divergence)
of the chosen stress tensor makes me not think this is particularally
relevant to the real world.

[...]

Eric Gisse

unread,
May 26, 2009, 12:18:33 PM5/26/09
to
On May 20, 8:06 am, Eric Gisse <jowr...@gmail.com> wrote:

[...]

An article on Doeleman's followup study was posted, in which this
arXiv article was referenced: http://arxiv.org/abs/0903.1105

It weaves together a lot of what I have been saying with more
technical details, and some stuff I did not know and/or had not
considered.

I do wish we had a greater grasp of the spin parameter, though. If we
had that, the accretion region's dynamics could be much more cleanly
understood.

Juan R.

unread,
May 27, 2009, 1:37:22 PM5/27/09
to
Eric Gisse wrote on Wed, 20 May 2009 16:06:12 +0000:

(...)

> I actually find it rather curious that the authors do not even
> seriously consider the RAIF model, which by the way is the generally
> accepted model, of accretion [ http://arxiv.org/abs/astro-ph/0304099]
> until midway through the paper in order to dismiss it using
> "observations" of magnetic fields that are based upon polarization
> of radio waves. That particular argument was not put together all
> that well, in my opinion.

One of the problems of the black hole model was to explain the
observed very small luminosity when the accretion rate is large. Some
authors suggested that the low luminosity could be explained by
introducing a new (untested) physical process named: RIAF (ADAF).

RIAF provides a kinetic energy model absorbtion by an hypotetical event
horizon forzing compatibility with observed luminosity. However, the
recently detected magnetic fields make RIAF practically impossible:

astro-ph/0308171

Bisnovatyi-Kogan G.S., Lovelace R.V.E., (2000), ApJ, 529, 978

You may enjoy the part of the ApJ article saying that previous works
"presented as a proof of the existence of the event horizon of black
holes" have no actual validity. Next some extracts:

"Some observational data that were interpreted as evidence for the
existence of the ADAF regime have disappeared after additional
accumulation of data. The most interesting example of this sort is
connected with the claim of "proof" of the existence of event
horizons of black holes due to manifestation of the ADAF regime of
accretion (Narayan, Garcia, & McClintock 1997). Analysis of a more
complete set of observational data (Chen et al. 1997) shows that the
statistical effect claimed as an evidence for ADAF disappears.
This example shows the danger of "proving" a theoretical model with
preliminary observational data. It is even more dangerous when the
model is physically not fully consistent, because then even a
reliable set of the observational data cannot serve as a proof of
the model."

Moreover, intrinsic magnetic fields have been
associated to many more other objects beyond SgrA*:

http://arxiv.org/abs/astro-ph/0505491

Observations Supporting the Existence of an Intrinsic Magnetic Moment
inside the Central Compact Object within the Quasar Q0957+561 2006:
ApJ, 132, 420-432. Rudolph E. Schild et al.

Direct Microlensing-Reverberation Observations of the Intrinsic
Magnetic Structure of Active Galactic Nuclei In Different Spectral
States: A Tale of Two Quasars 2008: ApJ, 135(1), 947-956. Rudolph E.
Schild et al

THE IMPACT OF NEUTRINO MAGNETIC MOMENTS ON THE EVOLUTION OF MASSIVE
STARS 2009: ApJ, 696, 608-619 Alexander Heger et al

Evidence for Intrinsic Magnetic Moments in Black Hole Candidates 2002:

ApJ 565, 447-454. Stanley L. Robertson et al.

http://arxiv.org/abs/astro-ph/0601662

http://arxiv.org/abs/astro-ph/0806.1748

(...)

>> Previously, I asked you to sustain a series of your claims and
>> acussations. You just avoided to reply.
>
> I actually forgot about this thread until you decided to start
> calling me a "crank" using this thread as a reference. Sore loser.

And again you avoided because you cannot. Now you pretend to hide
your unability in this newsgroup with one attempt to flamming, that
I will ignore :-D

(...)

> Does the so-called "renowned expert", whom I have never heard of,
> actually think his model is a serious contender or is it a toy model
> he is pursuing for his own particular reasons?

It does not matter if you never heard of him:

http://imprs-gw.aei.mpg.de/04_staff/luciano-rezzolla

http://www.aei.mpg.de/~rezzolla/Site/Home.html

(...)

> Class. Quan. Grav has published all sorts of interesting ideas over
> the years. The journal is not one that is primarily concerned with
> observation. That's for other journals like ApJ, Nature, etc. It is
> a journal for theoretical exercises - like MECO's.

The own Journal scope states that its scope includes
experimental gravitation (including strong field regime and tests of
gravitation theories)

http://www.iop.org/EJ/journal/-page=scope/0264-9381

http://www.iop.org/EJ/journal/-page=extra.5/0264-9381

(...)

> http://arxiv.org/abs/0709.0532
>
> Unstable for spin parameter >~ 0.4. Oops.
>
> http://www.iop.org/EJ/abstract/0004-637X/652/1/518
>
> "Based on a spectral analysis of the X-ray continuum that employs a
> fully relativistic accretion disk model, we conclude that the compact
> primary of the binary X-ray source GRS 1915+105 is a rapidly rotating
> Kerr black hole. We find a lower limit on the dimensionless spin
> parameter of a* > 0.98."
>
> OOPS.

First one opens the paper and finds that authors know the observational
data:

the binary X-ray source GRS 1915 + 105, which recent observations
identify as a rapidly-rotating object of spin a >= 0.98 M [6].

Their reference [6] is exactly the same ApJ you gave above. Surprise?
Then continue reading.

"Despite the wealth of circumstantial evidence, there is no definite
observational proof of the existence of astrophysical BHs. (A
review and a critique of current evidence can be found in Ref. [2]
and Ref. [8], respectively. See also Ref. [9] for a stimulating
mini-review.) Astrophysical objects without event horizon, yet
observationally indistinguishable from BHs, cannot be excluded a
priori."

They studied stability against scalar perturbations of systems with
ergoregions and no horizon. and gave "generic arguments suggesting"
what may be the scale for graviational perturbations. "Suggesting" or
"expecting" something is not the same than computing the scale.

Auhors probably aware of the limitations of their work concluded:

"Fast-spinning BH-like objects have been reported [6, 7]. The results
of this paper suggest that these objects must indeed be BHs."

Their "suggest [...] must inded be" is different from your belief
that objects were showed to be black holes.

The whole issue is still more interesting when the instability issue
has been recently revised in a more rigorous and complete way and found
*just* the contrary :-D

http://arxiv.org/abs/0808.4080

"Expanding on some recent results, we show that not all rotating
gravastars are unstable. Rather, stable models can be constructed also
with J/M^2 ∼ 1, [...] For the same reason, not all ultra-compact
astrophysical objects rotating with J/M^2 =< 1 are to be considered
necessarily black holes."

Thus, binary X-ray source GRS 1915 + 105 is perfectly compatible
with alternative models.

> http://arxiv.org/abs/0803.4200
>
> "Therefore, although the existence of gravastars cannot be excluded
> from such dynamical models, our results do indicate that, even if
> gravastars indeed exist, they do not exclude the existence of black
> holes."
>
> OOPS.

It seems they identify two 'phase-space' regions: one collapsing to
black hole and other does not. It is ironic this would imply that a
black hole collapse follows from you called a ridiculous model could
not explain the last 15 years of data :-D

Of course, you omited to cite the part of the abstract where they point
the existence of a phase space region that do not form a black hole.


--

Juan R.

unread,
May 27, 2009, 1:37:25 PM5/27/09
to
Eric Gisse wrote on Tue, 26 May 2009 16:18:33 +0000:

> On May 20, 8:06 am, Eric Gisse <jowr...@gmail.com> wrote:
>
> [...]
>
> An article on Doeleman's followup study was posted, in which this
> arXiv article was referenced: http://arxiv.org/abs/0903.1105

Doeleman et al. paper do not suport your belief that black holes have
been showed to exist.

http://www.nature.com/nature/journal/v455/n7209/abs/nature07245.html

In their paper they talk about "black holes candidate" and about the
"presumed black hole", because they did not show that you said.

Neither the above preprint support your belief that black holes have
been showed to exist. From authors conclusion:

"As a result, our conclusions may be applied more generally to all
gravitational theories that admit notions of energy conservation in
the test-particle limit. Specifically, these include all geometric
gravitational theories that admit stationary solutions, including
all of the f (R) theories and black hole alternatives that exist
within the context of GR. As a consequence, we cannot yet say that
Sgr A* is described by a GR black hole despite being able to conclude
that a horizon must exist."

Moreover, their claim they observed a horizon is open to criticism.
It seems they showed the existence of a "blackbody" surface:

"Consequently, high-redshift surfaces present a perverse realization
of the canonical pin-hole cavity, becoming ideal blackbodies as z
goes to infnity (Broderick & Narayan 2006). For a Schwarzschild
spacetime this is shown in Fig. 1; however this behavior is generic
to spherically symmetric spacetimes. Thus if the system has
sufficient time to have reached steady state, it must be a
blackbody."

And compare this with other models, as neutron stars, in the
section "Observational limits upon the existence of horizons"

"The primary astrophysical importance of a horizon is that the
gravitational binding energy liberated by material as it accretes
can be advected into the black hole without any further observational
consequence. This is very different from accretion onto other compact
objects, e.g., neutron stars, in which this liberated energy
ultimately must be emitted by the stellar surface. Importantly, this
argument is not dependent upon the particulars of the compact object.
Any object powered by accretion, whose surface is visible from the
external universe, should show evidence of surface radiation. We will
use this fact to rule out the possibility that accreted material in
Sgr A* settles in a region visible to outside observers, and in doing
so make the argument that a horizon /must/ exist.

The work is interesting because the quantum theory of gravitational
field (not GR) predicts existence of condensed objects (several
millions of solar masses) without singularities horizon but with
intrinsic magnetic field, and with the "high-redshift surfaces" that
authors observed (and interpreted as horizon).

However, the black hole model explanation of the low luminosity
phase is based in ADAF(RIAF) hypotesis, which is not compatible with
the observed magnetic fields (see references given in another message
in this same thread).

The quantum theory of gravitational field do not use ADAF(RIAF) and
explain the low luminosity phase by the "propeller effect" of the
magnetic field.. Thus it seems this model has better observational
support and is free of difficulties as singularities.


--

juanR...@canonicalscience.com

unread,
May 27, 2009, 9:28:32 PM5/27/09
to
Eric Gisse wrote on Fri, 22 May 2009 13:35:33 +0000:

> On May 21, 12:29 pm, juanREM...@canonicalscience.com wrote:
>
> [...]

Reintroducing link:

>>>>> http://arxiv.org/abs/0902.0346

>> Moreover, everyone can access the above pdf and find after their
>> equation (4.35):
>>
>> "However, the renormalised stress-energy-momentum tensor gives an
>> arbitrarily large (albeit finite) energy-condition-violating
>> contribution to the right hand side of the semiclassical Einstein
>> equations as the horizon formation condition 2M/r=1 is
>> approached."
>
> Key words _arbitrarily large_.

Key words "(albeit finite)". Your previous claim (sniped by you now) that
they gave "a stress tensor that diverges as one approaches the event
horizon" is plain false. The tensor do *not* diverges, it is *finite*.
They state this several times in the paper:

"everything is finite at an event horizon"

"and one remains with a finite contribution that depends on the details of
collapse"

"renormalised stress-energy-momentum tensor, although finite, could lead
to significant deviations from classical collapse when a trapping horizon
is just about to form"

And, of course, the first quote.

Moreover, authors of above pdf put their work in a more general
perspective and link it to other developments done in string theory, loop
quantum gravity, analogue spacetimes, etc. and give further references.

> In other words, it can be _whatever you want it to be_.

As stated before the value is computed from the model. Its value
will be given by the specific system under study, both by structural
parameter and the by details of the collapse (such as the speed of
collapse).

Eric Gisse

unread,
May 28, 2009, 12:53:52 PM5/28/09
to
On May 27, 9:37�am, "Juan R." Gonz�lez-�lvarez

<juanREM...@canonicalscience.com> wrote:
> Eric Gisse wrote on Tue, 26 May 2009 16:18:33 +0000:
>
> > On May 20, 8:06 am, Eric Gisse <jowr...@gmail.com> wrote:
>
> > [...]
>
> > An article on Doeleman's followup study was posted, in which this
> > arXiv article was referenced:http://arxiv.org/abs/0903.1105
>
> Doeleman et al. paper do not suport your belief that black holes have
> been showed to exist.

Actually, it does. Read the paper.

I continue to wonder where the goal post lies for black holes. How
strong must the evidence be?

>
> http://www.nature.com/nature/journal/v455/n7209/abs/nature07245.html

Yes, I know. I gave it to you.

>
> In their paper they talk about "black holes candidate" and about the
> "presumed black hole", because they did not show that you said.

You are relying on overly conservative but intrinstic scientific doubt
to make your case. Can you really not do better than that?

>
> Neither the above preprint support your belief that black holes have
> been showed to exist. From authors conclusion:

Holy crap. What does it take to make you reconsider?

>
> � "As a result, our conclusions may be applied more generally to all


> � gravitational theories that admit notions of energy conservation in
> � the test-particle limit. Specifically, these include all geometric
> � gravitational theories that admit stationary solutions, including
> � all of the f (R) theories and black hole alternatives that exist
> � within the context of GR. As a consequence, we cannot yet say that
> � Sgr A* is described by a GR black hole despite being able to conclude
> � that a horizon must exist."
>
> Moreover, their claim they observed a horizon is open to criticism.

Regardless, they made a strong case for the existence of a horizon.

> It seems they showed the existence of a "blackbody" surface:
>
> �"Consequently, high-redshift surfaces present a perverse realization
> � of the canonical pin-hole cavity, becoming ideal blackbodies as z
> � goes to infnity (Broderick & Narayan 2006). For a Schwarzschild
> � spacetime this is shown in Fig. 1; however this behavior is generic
> � to spherically symmetric spacetimes. Thus if the system has
> � sufficient time to have reached steady state, it must be a
> � blackbody."

Try reading the whole paper. No such high redshift surface was found.
The surface was, in fact, ruled out because of the near-100%
conversion efficiencies needed and the complete lack of observational
support in the IR spectrum.

>
> And compare this with other models, as neutron stars, in the
> section "Observational limits upon the existence of horizons"

Neutron stars have surfaces.

Neutron stars make optical noise when stuff falls on the surface.

Neutron stars are orders of magnitude less compact than what would be
required to explain the goings-on @ Sgr. A*.

>
> �"The primary astrophysical importance of a horizon is that the


> �gravitational binding energy liberated by material as it accretes
> �can be advected into the black hole without any further observational
> �consequence. This is very different from accretion onto other compact
> �objects, e.g., neutron stars, in which this liberated energy
> �ultimately must be emitted by the stellar surface.

This is important. You should read this paragraph again.

> Importantly, this
> �argument is not dependent upon the particulars of the compact object.

This is also important. The argumet is _independent_ from the
specifics of the alternatives so long as there _is_ an exterior but
highly redshfited surface.

This is why I consider the gravastar / MECO concepts to be ruled out
or at least made fantastically unlikely - without even considering
difficulties intrinstic to the objects themselves.

> �Any object powered by accretion, whose surface is visible from the


> �external universe, should show evidence of surface radiation. We will
> �use this fact to rule out the possibility that accreted material in
> �Sgr A* settles in a region visible to outside observers, and in doing
> �so make the argument that a horizon /must/ exist.
>
> The work is interesting because the quantum theory of gravitational
> field (not GR) predicts existence of condensed objects (several
> millions of solar masses) without singularities horizon but with
> intrinsic magnetic field, and with the "high-redshift surfaces" that
> authors observed (and interpreted as horizon).

You are now relying on a theory of gravitation that _does not exist_
in order to explain observation in order to avoid using classical
general relativity.

If you have an actual reference for this I'd love to see it but the
fact remains you are seriously grasping at straws here.

>
> However, the black hole model explanation of the low luminosity
> phase is based in ADAF(RIAF) hypotesis, which is not compatible with
> the observed magnetic fields (see references given in another message
> in this same thread).

The _inferred_ magnetic fields are not inconsistent with the RIAF
model unless you would like to make the claim that a magnetic field
inside a relativistic plasma is an unusual thing. Plus, if you read
your references, the _inferred_ magnetic fields are based on
assumptions by Robinson, Leiter, etc. _assuming_ the MECO concept is
correct.

The only argument that *could* be made is that highly variable
magnetic fields contradict the assumptions of the RIAF model, however
there is no evidence for that.

>
> The quantum theory of gravitational field do not use ADAF(RIAF) and
> explain the low luminosity phase by the "propeller effect" of the
> magnetic field.. Thus it seems this model has better observational
> support and is free of difficulties as singularities.

We'll see.

>
> --http://www.canonicalscience.org/
>
> Usenet Guidelines:http://www.canonicalscience.org/en/miscellaneouszone/guidelines.html


Eric Gisse

unread,
May 28, 2009, 12:53:56 PM5/28/09
to
On May 27, 9:37 am, "Juan R." González-Álvarez

<juanREM...@canonicalscience.com> wrote:
> Eric Gisse wrote on Wed, 20 May 2009 16:06:12 +0000:
>
> (...)
>
> > I actually find it rather curious that the authors do not even
> > seriously consider the RAIF model, which by the way is the generally
> > accepted model, of accretion [http://arxiv.org/abs/astro-ph/0304099]
> > until midway through the paper in order to dismiss it using
> > "observations" of magnetic fields that are based upon polarization
> > of radio waves. That particular argument was not put together all
> > that well, in my opinion.
>
> One of the problems of the black hole model was to explain the
> observed very small luminosity when the accretion rate is large. Some
> authors suggested that the low luminosity could be explained by
> introducing a new (untested) physical process named: RIAF (ADAF).

Notice how the discussion has shifted far away from the actual
observational evidence but rather onto the suitability of the current
models.

>
> RIAF provides a kinetic energy model absorbtion by an hypotetical event
> horizon forzing compatibility with observed luminosity. However, the
> recently detected magnetic fields make RIAF practically impossible:

Why, because a magnetic field inside of relativistic plasma is
unexpected?

>
> astro-ph/0308171

The conclusion is general to the point of being worthless, as the
specific situation isn't even analyzed. Such an argument could be
"applied" to solar and planetary plasma systems to argue that
electrons and protons don't have separate temperatures.

I reject the argument but the notion that the electrons should be
orders of magnitude cooler than the ions does strike me as odd,
however I am NOT an expert in plasma physics by any stretch of the
imagination.

>
> Bisnovatyi-Kogan G.S., Lovelace R.V.E., (2000), ApJ, 529, 978

Far more useful. I wonder how long it took you to find this stuff.

The paper assumes an optically _thin_ system, whereas the RIAF model
assumes an optically _thick_ system. Might be important...

The principle assumption here seems to be that the magnetic field of
the system is chaotic, even though there's no observational evidence
for/against that claim. At least, to my knowledge.

The accretion flow at Sgr. A* isn't spherical - it is more toroidal/
annular, which contradicts another assumption of the paper.
Furthermore, the RIAF model specifically assumes a geometrically
_thin_ accretion region.

Assuming a quasispherical accretion flow is ok, but doesn't survive
contact with observation. Especially when the paper mentions that the
radiative efficiency of a black hole could push as high as ~30%, while
observation puts it at a fraction of that.

It is specifically assumed that the effects from magnetic fields in
the form of reconnection is ignored in the RIAF model and I really
don't know how wrong that is. We have no detailed observations of the
accretion region to say it is right or wrong, except that the model
remains consistent on a broader scale with observation.

>
> You may enjoy the part of the ApJ article saying that previous works
> "presented as a proof of the existence of the event horizon of black
> holes" have no actual validity. Next some extracts:

No, the quote does not say that it has "no actual validity".

ASSSUMING the paper is correct, it is UNLIKELY that the RIAF model is
invalid.

[...repetition of paper's contents...]

> Moreover, intrinsic magnetic fields have been
> associated to many more other objects beyond SgrA*:
>
> http://arxiv.org/abs/astro-ph/0505491

So?

That's neat but quite irrelevant.

>
> Observations Supporting the Existence of an Intrinsic Magnetic Moment
> inside the Central Compact Object within the Quasar Q0957+561 2006:
> ApJ, 132, 420-432. Rudolph E. Schild et al.

-1, Wrong. The _Astronomical_ Journal is NOT ApJ.

Why are you citing a paper that has been repeatedly discussed
previously? There are no actual observations, or anything that can be
reasonably interpreted to be observations, of magnetic fields.

Even if there were, it is 100% consistent with a plasma. Plasmas
contain magnetic fields, I think I learned that one in my space
physics course before.

>
> Direct Microlensing-Reverberation Observations of the Intrinsic
> Magnetic Structure of Active Galactic Nuclei In Different Spectral
> States: A Tale of Two Quasars 2008: ApJ, 135(1), 947-956. Rudolph E.
> Schild et al

Not even looking at this one.

>
> THE IMPACT OF NEUTRINO MAGNETIC MOMENTS ON THE EVOLUTION OF MASSIVE
> STARS 2009: ApJ, 696, 608-619 Alexander Heger et al

WTF kind of scattershot is this? This is 100% irrelevant.

>
> Evidence for Intrinsic Magnetic Moments in Black Hole Candidates 2002:
> ApJ 565, 447-454. Stanley L. Robertson et al.

Notice the double standard. When Robertson, Leiter, etc, present a
model that is consistent with extremely sparse amounts of observation,
it is sufficient for you. But when I point to a case that has 15 years
of IR/microwave/optical surveys, nothing is ever enough.

Again - nothing written here is relevant. There are no observations of
magnetic moments, intrinstic or otherwise.

>
> http://arxiv.org/abs/astro-ph/0601662

Simply wrong. There is no stable configuration of matter that lies
within its' own evenr horizon.

>
> http://arxiv.org/abs/astro-ph/0806.1748

I tire of the endless citation of Leiter. It is the same old song
every time and I'm done discussing it.
[...]

> > Does the so-called "renowned expert", whom I have never heard of,
> > actually think his model is a serious contender or is it a toy model
> > he is pursuing for his own particular reasons?
>
> It does not matter if you never heard of him:
>
> http://imprs-gw.aei.mpg.de/04_staff/luciano-rezzolla
>
> http://www.aei.mpg.de/~rezzolla/Site/Home.html

Answer the question. Does he think it is a serious contender, or a toy
model?

[...]

> >http://arxiv.org/abs/0709.0532
>
> > Unstable for spin parameter >~ 0.4. Oops.
>
> >http://www.iop.org/EJ/abstract/0004-637X/652/1/518
>
> > "Based on a spectral analysis of the X-ray continuum that employs a
> > fully relativistic accretion disk model, we conclude that the compact
> > primary of the binary X-ray source GRS 1915+105 is a rapidly rotating
> > Kerr black hole. We find a lower limit on the dimensionless spin
> > parameter of a* > 0.98."
>
> > OOPS.
>
> First one opens the paper and finds that authors know the observational
> data:
>
>   the binary X-ray source GRS 1915 + 105, which recent observations
>   identify as a rapidly-rotating object of spin a >=  0.98 M [6].
>
> Their reference [6] is exactly the same ApJ you gave above. Surprise?

Notice how they not only cite it, but agree with it.

> Then continue reading.
>
>   "Despite the wealth of circumstantial evidence, there is no definite
>   observational proof of the existence of astrophysical BHs. (A
>   review and a critique of current evidence can be found in Ref. [2]
>   and Ref. [8], respectively. See also Ref. [9] for a stimulating
>   mini-review.) Astrophysical objects without event horizon, yet
>   observationally indistinguishable from BHs, cannot be excluded a
>   priori."

Were it not for your valiant efforts I would never have been able to
see the words of the paper I read and then cited.

The question is the same as it is before: WHAT DOES IT TAKE FOR YOU TO
EVEN CONSIDER THE POSSIBILITY?

Every observation is consistent with general relativistic black holes.
Every single one. The only counter you have are increasingly contrived
alternatives that do not even fit observation, eg: MECO/gravastar.

>
> They studied stability against scalar perturbations of systems with
> ergoregions and no horizon. and gave "generic arguments suggesting"
> what may be the scale for graviational perturbations. "Suggesting" or
> "expecting" something is not the same than computing the scale.

Whereas vague arguments against the RAIF model are perfectly OK
because it supports your predetermined conclusion.

>
> Auhors probably aware of the limitations of their work concluded:
>
> "Fast-spinning BH-like objects have been reported [6, 7]. The results
> of this paper suggest that these objects must indeed be BHs."
>
> Their "suggest [...] must inded be" is different from your belief
> that objects were showed to be black holes.

Do you disagree with the author's work?

Ok.

Do you have an alternative model that allows for near-extremal
rotation that _isn't_ a black hole and _doesn't_ have an event
horizon?

Interesting.

>
> The whole issue is still more interesting when the instability issue
> has been recently revised in a more rigorous and complete way and found
> *just* the contrary :-D
>
> http://arxiv.org/abs/0808.4080

I find it curious to see how studiously you defend the gravastar
concept despite itself being a far more silly concept than the black
hole. Layers of deSitter space, infinitely thin shell of matter, eh?
Rather reminds me of the increasinly contrived alternatives some folks
invoke to explain relativity.

Other fun features of the concept:

* A material speed of sound equivalent to the speed of light.

* There is NO known interior solution to the Kerr metric. At least
that I have ever found. This makes the matching up of a static deSiter
interior against a rotating exterior to be....interesting.

* The paper concludes that a spin parameter larger than 1 is possible.
It would be a simple and rather definitive result were J to ever be
found to be >= 1, except that has never been seen. Perhaps that's just
a coincidence...?

I am yet to hear any motivating physical arguments for this concept.
Like how the hell it is supposed to form.

>
> "Expanding on some recent results, we show that not all rotating
> gravastars are unstable. Rather, stable models can be constructed also
> with J/M^2 ∼ 1, [...] For the same reason, not all ultra-compact
> astrophysical objects rotating with J/M^2 =< 1 are to be considered
> necessarily black holes."
>
> Thus, binary X-ray source GRS 1915 + 105 is perfectly compatible
> with alternative models.

While being perfectly compatible with the standard model without
having the albatross of being physically ridiculous surrounding it.
The process of forming a black hole has been extensively investigated,
both analytically and numerically.

>
> >http://arxiv.org/abs/0803.4200
>
> > "Therefore, although the existence of gravastars cannot be excluded
> > from such dynamical models, our results do indicate that, even if
> > gravastars indeed exist, they do not exclude the existence of black
> > holes."
>
> > OOPS.
>
> It seems they identify two 'phase-space' regions: one collapsing to
> black hole and other does not. It is ironic this would imply that a
> black hole collapse follows from you called a ridiculous model could
> not explain the last 15 years of data :-D

Except GR black holes _do_ explain observation with a much more
physically sound and consistent fashion than the alternatives you put
forth.

>
> Of course, you omited to cite the part of the abstract where they point
> the existence of a phase space region that do not form a black hole.

Most likely because I assumed you were capable of reading. The point
of the reference was that black holes are entirely consistent with
observation even _if_ the gravastar is assumed.

>
> --http://www.canonicalscience.org/
>
> Usenet Guidelines:http://www.canonicalscience.org/en/miscellaneouszone/guidelines.html


Juan R.

unread,
May 28, 2009, 5:40:36 PM5/28/09
to
"Juan R." González-Álvarez wrote on Wed, 27 May 2009 19:37:22 +0200:

(...)

> http://arxiv.org/abs/astro-ph/0806.1748

Sorry, it is

http://arxiv.org/abs/0806.1748

Juan R.

unread,
Jun 12, 2009, 1:03:02 PM6/12/09
to
Eric Gisse wrote on Thu, 28 May 2009 18:53:52 +0200:

> On May 27, 9:37 am, "Juan R." Gonz�lez-�lvarez
> <juanREM...@canonicalscience.com> wrote:
>> Eric Gisse wrote on Tue, 26 May 2009 16:18:33 +0000:
>>
>> > On May 20, 8:06 am, Eric Gisse <jowr...@gmail.com> wrote:
>>
>> > [...]
>>
>> > An article on Doeleman's followup study was posted, in which this
>> > arXiv article was referenced:http://arxiv.org/abs/0903.1105
>>
>> Doeleman et al. paper do not suport your belief that black holes have
>> been showed to exist.

(...)

But you continue to cite it as if were supporting your own claims. Below
again you disagree with authors are really saying.

>> In their paper they talk about "black holes candidate" and about the
>> "presumed black hole", because they did not show that you said.
>
> You are relying on overly conservative but intrinstic scientific doubt
> to make your case. Can you really not do better than that?

Your misunderstand of the paper and your fail to grasp *why* the authors
continue to use the terms "black holes candidate" and "presumed black
hole" in their work, are not valid excuses for your continued pretension
this reference is supporting your own ideas.

I talk about black hole candidates, they talk about black hole candidates.
We disagree with you.

>> Neither the above preprint support your belief that black holes have
>> been showed to exist. From authors conclusion:
>
> Holy crap. What does it take to make you reconsider?

Again you dishonestly pretended to cite a preprint as if authors were
supporting your rants and beliefs, and when I have pointed it is just the
contrary and when I have quoted author conclusions. You *snip* them and
qualify as "holy crap".

If you do not agree with they writing about how "black hole alternatives"
explain the same data or if you do not like their claim "we cannot yet say
that Sgr A* is described by a GR black hole" because contradicts your
pretension, that is your problem.

(...)

>> It seems they showed the existence of a "blackbody" surface:
>>
>> �"Consequently, high-redshift surfaces present a perverse
>> realization of the canonical pin-hole cavity, becoming ideal
>> blackbodies as z goes to infnity (Broderick & Narayan 2006). For a
>> Schwarzschild spacetime this is shown in Fig. 1; however this
>> behavior is generic to spherically symmetric spacetimes. Thus if the
>> system has sufficient time to have reached steady state, it must be a
>> blackbody."
>
> Try reading the whole paper. No such high redshift surface was found.
> The surface was, in fact, ruled out because of the near-100% conversion
> efficiencies needed and the complete lack of observational support in
> the IR spectrum.

They studied a surface (yes subindex "surf" in expressions 4, 5, 9, 10...
and in figures 4 and 5... mean "surface" even if you did not noticed) and
found that surface behaves like they named a blackbody surface.

They also note in several parts of the preprint that a very high z surface
is equivalent to a perfect blackbody surface, e.g. in figure 1, in page
3...

(...)

(rest sniped)


--

Juan R. González-Álvarez

unread,
Jun 12, 2009, 1:23:43 PM6/12/09
to
[[Mod. note -- A number of recent posts in this thread are coming
quite close to breaching the newsgroup charter's prohibition on
"Personal attacks (i.e. flames)". Please, everyone, let's try to
focus on the *physics*. There seem to be enough points of disagreement
there to keep us busy for a while...
-- jt]]

Eric Gisse wrote on Thu, 28 May 2009 18:53:56 +0200:

> On May 27, 9:37 am, "Juan R." González-Álvarez


> <juanREM...@canonicalscience.com> wrote:
>> Eric Gisse wrote on Wed, 20 May 2009 16:06:12 +0000:

(...)

> Notice how the discussion has shifted far away from the actual


> observational evidence but rather onto the suitability of the current
> models.

I notice that you wrote:

"I actually find it rather curious that the authors do not even

seriously consider the RAIF model".

You were explained *why* they did not use ADAF(RAIF) and it was also
explained to you why the use of ADAF would give an inconsistent
model:

"It is even more dangerous when the model is physically not fully
consistent, because then even a reliable set of the observational data

cannot serve as a proof of the model."

(...)

>> astro-ph/0308171
>
> The conclusion is general to the point of being worthless, as the
> specific situation isn't even analyzed. Such an argument could be
> "applied" to solar and planetary plasma systems to argue that electrons
> and protons don't have separate temperatures.

No, the author does clear in what situations ADAF works.

(...)



>> Bisnovatyi-Kogan G.S., Lovelace R.V.E., (2000), ApJ, 529, 978

(...)

> The paper assumes an optically _thin_ system, whereas the RIAF model
> assumes an optically _thick_ system. Might be important...

Wrong, optically thin RIAF is invoked to explain how the supermassive BH
candidates activity switchs off while the accretion rate decreases.

E.g. it was claimed in literature that the spectrum of Sgr A* was
explained by the optically thin advection dominated accretion flow model.

Bisnovatyi-Kogan & Lovelace paper showed this kind of models to be
inconsistent.

> The principle assumption here seems to be that the magnetic field of the
> system is chaotic, even though there's no observational evidence
> for/against that claim. At least, to my knowledge.

No assumption here. Moreover, they link with observations of chaotic
coronal magnetic field and others obs.

> The accretion flow at Sgr. A* isn't spherical - it is more toroidal/
> annular, which contradicts another assumption of the paper.

(i)
Wrong. Authors studied a quasi-spherical flow.

(ii)
Moreover, it was in BH physics were spherical models were assumed. E.g.
when Melia claimed that Sgr A* was a BH in his:

An accreting black hole model for Sagittarius A 1992: ApJ 387, L25. Melia,
F.

Not just you invent again assumptions are not in the papers you dislike
but you ignore your arguments when contradict the papers you like. This is
not serious.

(...)

> It is specifically assumed that the effects from magnetic fields in the
> form of reconnection is ignored in the RIAF model and I really don't
> know how wrong that is.

Wrong. It is not their assumption but one of the assumptions used in RIAF
model.

>> You may enjoy the part of the ApJ article saying that previous works
>> "presented as a proof of the existence of the event horizon of black
>> holes" have no actual validity. Next some extracts:
>
> No, the quote does not say that it has "no actual validity".

You sniped the quote explaining how previous evidence dissappeared:

"Some observational data that were interpreted as evidence for the
existence of the ADAF regime have disappeared after additional
accumulation of data. The most interesting example of this sort is
connected with the claim of "proof" of the existence of event horizons
of black holes due to manifestation of the ADAF regime of accretion
(Narayan, Garcia, & McClintock 1997). Analysis of a more complete set of
observational data (Chen et al. 1997) shows that the statistical effect
claimed as an evidence for ADAF disappears. This example shows the
danger of "proving" a theoretical model with preliminary observational
data. It is even more dangerous when the model is physically not fully
consistent, because then even a reliable set of the observational data

cannot serve as a proof of the model."

(...)

>> Observations Supporting the Existence of an Intrinsic Magnetic Moment
>> inside the Central Compact Object within the Quasar Q0957+561 2006:
>> ApJ, 132, 420-432. Rudolph E. Schild et al.
>
> -1, Wrong. The _Astronomical_ Journal is NOT ApJ.

Agree and your only contribution to this thread. The reference is

Observations Supporting the Existence of an Intrinsic Magnetic Moment
inside the Central Compact Object within the Quasar Q0957+561 2006:

Astron.J, 132, 420-432. Rudolph E. Schild et al.

(...)

>> http://arxiv.org/abs/astro-ph/0601662
>
> Simply wrong. There is no stable configuration of matter that lies
> within its' own evenr horizon.

Nonsense. There is not even horizon in the paper "Sgr A* as probe of the
theory of supermassive compact objects without event horizon".

Search in a dictionary the meaning for "without".

(...)

>> > Does the so-called "renowned expert", whom I have never heard of,
>> > actually think his model is a serious contender or is it a toy model
>> > he is pursuing for his own particular reasons?
>>
>> It does not matter if you never heard of him:
>>
>> http://imprs-gw.aei.mpg.de/04_staff/luciano-rezzolla
>>
>> http://www.aei.mpg.de/~rezzolla/Site/Home.html
>
> Answer the question. Does he think it is a serious contender, or a toy
> model?

This is a very pretty ridiculous question, to divert readers away from
central issue that your accusations he did not understand BHs or that he
was ignoring observations were without basis.

Unlike you, he is an expert in black holes. He knows very well the models,
whereas you are showing your 'knowledge' here.

> [...]
>
>> >http://arxiv.org/abs/0709.0532
>>
>> > Unstable for spin parameter >~ 0.4. Oops.
>>
>> >http://www.iop.org/EJ/abstract/0004-637X/652/1/518
>>
>> > "Based on a spectral analysis of the X-ray continuum that employs a
>> > fully relativistic accretion disk model, we conclude that the compact
>> > primary of the binary X-ray source GRS 1915+105 is a rapidly rotating
>> > Kerr black hole. We find a lower limit on the dimensionless spin
>> > parameter of a* > 0.98."
>>
>> > OOPS.
>>
>> First one opens the paper and finds that authors know the observational
>> data:
>>

>>   the binary X-ray source GRS 1915 + 105, which recent observations  
>> identify as a rapidly-rotating object of spin a >=  0.98 M [6].


>>
>> Their reference [6] is exactly the same ApJ you gave above. Surprise?
>
> Notice how they not only cite it, but agree with it.

This is that means my above "authors know the observational data". Your
repetition of I have said is unuseful again.

It seems you are only submitting either flagrant misreadings of papers
contradicting you or just repeating I am saying.

>> Then continue reading.
>>
>>   "Despite the wealth of circumstantial evidence, there is no
>> definite observational proof of the existence of astrophysical BHs.
>> (A review and a critique of current evidence can be found in Ref. [2]
>> and Ref. [8], respectively. See also Ref. [9] for a stimulating
>> mini-review.) Astrophysical objects without event horizon, yet
>> observationally indistinguishable from BHs, cannot be excluded a
>> priori."
>
> Were it not for your valiant efforts I would never have been able to see
> the words of the paper I read and then cited.

You cited it as if was supporting your invalid ideas, and arrogantly wrote
your "Oops" and "OOPS", still quoted above.

I merely quoted a part of the reference you introduced where it says the
contrary than you.

(...)



>> The whole issue is still more interesting when the instability issue
>> has been recently revised in a more rigorous and complete way and found
>> *just* the contrary :-D
>>
>> http://arxiv.org/abs/0808.4080
>
> I find it curious to see how studiously you defend the gravastar concept
> despite itself being a far more silly concept than the black hole.

This ad hominem is kind of scary. You *said* that the model was "Unstable
for spin parameter >~ 0.4. Oops." and I have showed, with above preprint,
that the model is stable and explains binary X-ray source GRS 1915 + 105
for the which >= 0.98.

What would I do? To say "yes, Eric it is unstable", when it is not?

(...)

>> >http://arxiv.org/abs/0803.4200

(...)

>> > OOPS.
>>
>> It seems they identify two 'phase-space' regions: one collapsing to
>> black hole and other does not. It is ironic this would imply that a
>> black hole collapse follows from you called a ridiculous model could
>> not explain the last 15 years of data :-D
>
> Except GR black holes _do_ explain observation with a much more
> physically sound and consistent fashion than the alternatives you put
> forth.

Not just you have not proved this, but you are sistematically ignored any
reference contradicting you, including the own references you cited and
that state in clear form that data is compatible with alternatives, as can
be seen in some of the multiple quotes.

0 new messages